[ /tv/ /rf/ /vg/ /a/ /b/ /u/ /bo/ /fur/ /to/ /dt/ /cp/ /oe/ /bg/ /ve/ /r/ /mad/ /d/ /mu/ /cr/ /di/ /sw/ /hr/ /wh/ /lor/ /s/ /hau/ /slow/ /gf/ /vn/ /w/ /ma/ /azu/ /wn/ ] [ Main | Settings | Bookmarks | Music Player ]

No.151715 Reply
File: Теорема-Гильберта-о-корнях.-Или-о-нулях.jpg
Jpg, 158.84 KB, 1366×768 - Click the image to expand
edit Find source with google Find source with iqdb
Теорема-Гильберта-о-корнях.-Или-о-нулях.jpg
Наша кафедра продолжает свою работу. Здесь мы обсуждаем математику, а также иногда отвечаем на простые вопросы доброанонов.
Тред обучения математике: >>124265

Библиотека: http://gen.lib.rus.ec
Способ читать научные статьи: https://sci-hub.io
Ответы на вопросы: http://mathoverflow.net

Калькуляторы:
http://online-integral.ru
http://www.wolframalpha.com

Предыдущий: >>144684
>> No.151722 Reply
File: faceКотскKindaPepeSadFrog14544127191980.jpg
Jpg, 49.20 KB, 628×676 - Click the image to expand
edit Find source with google Find source with iqdb
faceКотскKindaPepeSadFrog14544127191980.jpg
Перекачу нерешённой задачкой: верно ли что если для группы G все простые G-модули над алгебраически замкнутым полем одномерны, то она абелева?
>> No.151723 Reply
Анон как гуманнитарию 26лет отроду, по образованию художнику, суметь стать физиком-математиком, восзможно ли это вообще?
>> No.151724 Reply
>>151723
Кто знает. По крайней мере можешь попробовать.
>> No.151725 Reply
>>151723
Хочешь 4-6 лет ходить в вуз, чтобы выпуститься с бэкграндуом 22-летнего выпускника раздолба и с куда менее пластичными мозгами?
...но зачем?
>> No.151726 Reply
>>151725
Такая вот мания.Что поделать.
Самому то ее еще сложнее осилить?
>> No.151727 Reply
>>151726
Да, весьма непросто.
>> No.151729 Reply
Порекомендуйте тогда самого лучшего.Как изи, так и медиум и хард.
>> No.151730 Reply
>>151722
Не могу решить в лоб. Надо подумать.
>> No.151742 Reply
>>151726
> Самому то ее еще сложнее осилить?
Здесь сложно давать советы, потому что вопрос упирается скорее в твою мотивацию (которая мне откровенно непонятна) количество времени, которое ты готов посвятить и т. д. и т. п.

Походить и послушать в НМУ можно в любом возрасте, редко это выливается во что-то большее, чем мимолётное ощущение собственно превсходства от того, что нахватался каких-то вещей по верхушкам.
>> No.151749 Reply
Короче, учусь в топтехновузе на инженера. Поступал с надеждой совместить математику с прикладными навыками, но прогадал. Так вот, антоний, учиться мне ещё несколько лет, и как думаешь, поступать ли в аспирантуру на математику при условии, что смогу туда поступить? Есть ли смысл, как думаешь?
>> No.151751 Reply
>>151749
Есть, если хочется.
>> No.151808 Reply
File: -.jpg
Jpg, 273.94 KB, 600×600 - Click the image to expand
edit Find source with google Find source with iqdb
-.jpg
Не могу сделать домашку, нужно написать доказательства связанные со статистикой, хотя я не математик. Там всякие функции стоимости и unbiased estimators. Много-много добра тому кто мне поможет, ну и всем подвернувшимся.
neko@techie.com
>> No.151811 Reply
>>151723
Бери и учись, в чём проблема?
>> No.151947 Reply
Пожалуйста, помогите построить кратчайшую сеть Штейнера для куба. Вообще не секу как делать. Доказывать минимальность не обязательно, нужна лишь сама сеть. Спасибо !
>> No.151949 Reply
>>151808
О, так это ты что ли в треде про нейросети писал.
>> No.151950 Reply
Доброанон, распиши, пожалуйста, покомпонентно $[\partialX, \partialY] f$ (коммутатор производных вдоль векторных полей). Я уже вторые сутки не могу понять, почему это, во-первых, дифференциальный оператор первого (а не второго порядка), а, во-вторых, почему всё просто не сокращается? Безмерно туп и мне стыдно, надеюсь на вас.
>> No.151955 Reply
>>151715
> простые вопросы доброанонов.
Теперь я знаю кого долбить моими лёгкими задачками.
мимоубежал
>> No.151961 Reply
>>151947
Я за сутки ничего не надумал, туго с головой. Помогите, пожалуйста.
>> No.151964 Reply
>>151950
Разобрался, вопрос снимается.
>> No.151979 Reply
File: W511N.png
Png, 30.90 KB, 556×412 - Click the image to expand
edit Find source with google Find source with iqdb
W511N.png
>>151961
Я кретин, не додумался поискать в англоязычном сегменте. А там есть эта инфа. Если кому интересно - вот.
>> No.152087 Reply
Почему считать надо именно квадратичное отклонение?
>> No.152088 Reply
>>152087
Мимореквестирую [ссылку на] ответ на этот вопрос на пальцах для человека, который уже пытался понять, но не до конца понял, чем оно лучше других степеней.
>> No.152096 Reply
>>152087
В какой задаче?
>> No.152100 Reply
Ни в какой. Хочется понять, почему именно квадратичное отклонение используют в качестве меры разброса случайной величины, и получается очень даже неплохо.
>> No.152113 Reply
>>152100
Что значит "В качестве меры разброса"?
В качестве оценки дисперсии? Ну дык, википедия → «выборочная дисперсия»

В качестве функции потерь? Ну дык, это не во всях методах/алгоритмах. Вот AdaBoost, к примеру, минимизирует экспоненциальную функцию потерь и отлично работает.

В некоторых задачах при некоторых допущениях квадратичная функция потерь имеет математический смысл.

В частности, оценки, полученные методом наименьших квадратов в случае гауссовой модели (точки лежат на линии/плоскости + нормально распределённая ошибка) являются оценками максимального правдоподобия.

А это влечёт за собой определённые плюшки (омп оценки состоятельны, асимптотически нормальны и асимптотически эффективны).
>> No.152188 Reply
Я полностью подавлен. Вот уже месяц я пытаюсь пройти курс элементарной алгебры, но я постоянно опускаю руки, двигался очень медленно, а сейчас в итоге вообще застрял. Из предложенных учебником задач, решал от 20 до 90 %, но никогда не смог прорешивать все, что накладывало свой отпечаток. В итоге сейчас застрял на квадратных уравнениях, да, не надо сразу думать, что я не могу найти дискриминант, но примеры, с которыми я сталкиваюсь, ставят меня в угол. Возможно я идиот. Занимаюсь по самому пособию, которое считается самым легким. Туманов "Элементарная алгебра". Материал понимаю, но часто не могу применить его для примеров сильно отличающихся по структуре от разобранных в главе.
>> No.152209 Reply
File: Dizzee-Rascal-Bassline-Junkie.png
Png, 754.87 KB, 1041×686 - Click the image to expand
edit Find source with google Find source with iqdb
Dizzee-Rascal-Bassline-Junkie.png
>>152087
Среднее значение не всегда отражает реальное положение вещей из-за отсутствия инфы о разбросе настоящих значений. Для компенсации этого недостатка вводят дисперсию. Дисперсия позволяет указать интервал [сред.знач-корень из диспер, сред.знач+корень из диспер]. С вероятность 65% реальные значения будут лежать в этом интервале. Если интервал расширить в три раза (метод трех сигм), то вероятность будет 98% с хуем или что-то близкое к 100, я не помню. Собственно среднеквадратичное отклонение и есть корень из дисперсии. Вычислить это отклонение легко и удобно. Собственно оно именно квадратное, потому что надо брать модуль разницы реальных значений и средних значений. Но модуль - это геморой и говно. Поэтому просто возводят в квадрат. Возможно, в моем ответе есть ложка вранья, потому что термины я не все помню, но суть именно в этом. Для подробной инфы можешь посмотреть какую-нибудь методичку о статистической обработке данных.
>> No.152216 Reply
>>152209
> Среднее значение не всегда отражает реальное положение вещей
"Среднее значение" иногда не значит вообще ничего. Например, когда математическое ожидание не существует.
Кстати, второй момент и дисперсия тоже существуют не всегда.

> С вероятность 65% реальные значения будут лежать в этом интервале.
Ты забыл сказать «Для нормально распределённой случайно величины». Для других величин это может быть неверным.

> Собственно среднеквадратичное отклонение и есть корень из дисперсии
из выборочный дисперсии просто, чтобы у читатель не запутался

> Собственно оно именно квадратное, потому что надо брать модуль разницы реальных значений и средних значений. Но модуль - это геморой и говно.

Цитирую википедию (которая в свою очередь цитирует умную книжку): «Среднее абсолютное отклонение использовалось в качестве оценки отклонения в исследовании операций на заре развития вычислительной техники, так как требовало меньших затрат вычислительных ресурсов по сравнению с более целесообразным среднеквадратическим отклонением».
>> No.152218 Reply
File: tumblr_o0wasiozSu1ruokcoo1_540.jpg
Jpg, 103.95 KB, 540×685 - Click the image to expand
edit Find source with google Find source with iqdb
tumblr_o0wasiozSu1ruokcoo1_540.jpg
>>152216
Спасибо, что поправил.
>> No.152426 Reply
Гипотезу Римана доказали.
инсайдер
>> No.152427 Reply
>>152426
Да быть не может.
>> No.152966 Reply
Посоветуйте cheat sheet по трюкам для инт./диф. калькулюса, чтобы быстро и легко решать задачи по матану
>> No.152969 Reply
>>152966
Wolframalpha — лучший cheat sheet.
>> No.152972 Reply
>>152969
Кстати, что-то он в последнее время через раз работает и дико тормозит. У меня одного такое?
>> No.153083 Reply
File: anon-gif-залипалово-2790675.gif
Gif, 1788.45 KB, 500×500 - Click the image to expand
edit Find source with google Find source with iqdb
anon-gif-залипалово-2790675.gif
Что такое дифференциал? Тот, который ставят в конце интеграла, а не тот, который в механике, конечно.
>> No.153085 Reply
>>153083
Зорич, Математический анализ.
Винберг, Курс алгебры.
Ленг, Алгебра.
Шварц, Анализ.
Спивак, Анализ на многообразиях.
>> No.153099 Reply
>>152087
> Почему считать надо именно квадратичное отклонение?
Ну там вариантов не много. Собственно, проще только абсолютное отклонение (по модулю). Проблема с ним возникает при оптимизации, например, при проведении регрессии: в общем случае у квадратичного отклонения один минимум, у абсолютного - бесконечно много.
>> No.153119 Reply
File: -.jpg
Jpg, 12.67 KB, 275×229 - Click the image to expand
edit Find source with google Find source with iqdb
-.jpg
Дорогая маткафедра. Я хочу запилить систему определения направления звука, с помощью группы микрофонов.
Суть проста — установка оной на компактного робота с целю следования последнего за источником звука. Предполагается группа из двух и более микрофонов, которые бы реагировали на фронт волны поочерёдно.
Исходя из приложенной халтуры (http://rgho.st/6TZbt45gW), откопанной мной в гугле, можно невероятным образом восстановить угол между осью микрофонов и направлением, откуда исходит звуковой сигнал.
Не мог бы мат-анон пояснить мне за принципы описанные в документе? Интересует, собственно, восстановление угла.
>> No.153197 Reply
>>153099
Почему не любое чётное?
>> No.153377 Reply
File: center.png
Png, 33.63 KB, 299×515 - Click the image to expand
edit Find source with google Find source with iqdb
center.png
Как называется точка О?
>> No.153378 Reply
>>153377
Вершина, ваш Кэп
>> No.153380 Reply
>>153377
Можешь назвать её "Виктор", в честь меня.
>> No.153381 Reply
>>153197
Потому что 2 - наименьшее из неотрицательных четных. Но можно любое. Можно и модуль от нечетных брать. Можно использовать хитровыебаные функции. Но квадрат - самое простое.
>> No.153385 Reply
>>152969
Дык
Нужно для техношараги, в которой нужно решать демидовича руками
>> No.153393 Reply
>>153377
Точка Торричелли.

>>153380
"Ваганыч".
>> No.153418 Reply
Как отличаются взгляды на логику и обоснование математики у Гильберта и Пуанкаре?
>> No.153419 Reply
>>153418
Уже никак, оба мертвы. Пуанкаре изначально был сторонником Кантора, позднее интересовался интуиционизмом. Над Пеано смеялся, над Бурали-Форти издевался. Его взгляды изложены в его книге "О науке". Ну а Гильберт изобрёл формальный метод, как известно.
>> No.153420 Reply
>>153419
Пожалуйста подробнее.
Почему Пуанкаре перешел в интуиционизм, какой здравый смысл в этом состоял, и что он не понимал по мнению Гильберта?
>> No.153422 Reply
>>153420
У меня уже есть подозрения, что расхождения между ними уже должны быть менее заметными, чем между например, Фреге и Гильбертом. Но почему последний сказал, что не нуждается в гипотезах Пуанкаре об интуиции? Он с чем-то несогласен, или это просто безразличие, вызванное различием интересов?
>> No.153448 Reply
File: DSC_0495.JPG
Jpg, 2386.11 KB, 3840×2160 - Click the image to expand
edit Find source with google Find source with iqdb
DSC_0495.JPG
Ваззап доброанон. Подскажи плес правильно ли я нашел верные и сомнительные числа, числа на пике
>> No.153449 Reply
Есть вопрос. При определении сомнительных чисел, с каким числом сравнивать значение абсолютной погрешности? Где то пишут что брать единицу разряда, по сканави - аж 5 ебучих единиц. Кому верить то? Помоги анон
>> No.153452 Reply
>>153448
Дай определения этим понятиям. Я не знаю, что такое "сомнительное число".
>> No.153454 Reply
>>153452
В приближенном числе есть такое понятие значимые числа, пишут что по ним можно судить о погрешности приближенного числа. Значимые числа в десятичной дроби это всякое число, начиная с первой ненулевой цифры слева направо и нули стоящие между этими цифрами. Значимые цифры делятся на верные и сомнительные. Какая цифра верная а какая нет определяется абсолютной погрешностью данного приближенного числа. В учебнике сканави написано что:

цифра из приближенного числа будет верной если абс погрешность не превосходит пяти единиц разряда следующего за этой цифрой если же превосходит то цифра и все что за ней - сомнительные

А многочисленные данные из интернетов, говорят что:
цифра из приближенного числа считается верной если абс погрешность числа не превышает единицы разряда данной цифры и это блядь только в широком смысле, в узком смысле цифра считается верной если абс погрешность не превышает единицы разряда следующего за цифрой
У сканави про узкий и широкий смысл нет нихуя.

Короч я так понял сомнительная цифра в числе, это та цифра которую можно округлить и погрешность от её округления добавить в абсолютную. Вот только никак не могу разобраться со способом определения сомнительных цифр в числе. Есть дробь например 236,563046 и есть погрешность 0,023. Как определить какие цифры в дроби верные, какие сомнительные?
>> No.153476 Reply
>>153454
> погрешность 0,023
Погрешность всегда округляется до первой значащей цифры, либо до двух, если эта самая значащая цифра "единица".
0.023 -> 0.02
0.51 -> 0.5
0.0192 -> 0.02
0.0121 -> 0.012
Думаю понял схему. Потом саму дробь округляешь до того знака, на котором остановился в погрешности. В твоем примере число будет 236.56, а погрешность 0.02. Знаки 236.5 точно определены, а шестерочка последняя сомнительная.
>> No.153663 Reply
Привет, мнимые анонимусы. Скажите, существует в природе учебный материал по дискретной математике, чтобы поменьше воды и букв и побольше сути? Желательно на русском.
>> No.153664 Reply
File: 10202.jpg
Jpg, 104.53 KB, 300×418 - Click the image to expand
edit Find source with google Find source with iqdb
10202.jpg
>>153663
Да, есть. Только осторожнее, в переводе имеются опечатки. Отношение частичного порядка на с.103 определено неверно, например.
>> No.153677 Reply
File: C__Data_Users_DefApps_AppData_INTERNETEXPLORER_Tem.jpg
Jpg, 46.33 KB, 604×402 - Click the image to expand
edit Find source with google Find source with iqdb
C__Data_Users_DefApps_AppData_INTERNETEXPLORER_Tem.jpg
>>153664
Вот спасибо! Держи Джона Рэмбо.
>> No.153911 Reply
File: 14553048499710.jpg
Jpg, 241.11 KB, 800×1184
edit Find source with google Find source with iqdb
14553048499710.jpg
File: 14616918923570.jpg
Jpg, 57.90 KB, 629×633
edit Find source with google Find source with iqdb
14616918923570.jpg

Господа Аноны. Узнал, что существует некий доброчановский список литературы по математике. Хотел бы его увидеть и сравнить с фочановским/краутчановским и программой Вербицкого. Реквестую этот список.
>> No.153912 Reply
>> No.153921 Reply
>>153911
Есть как минимум три списка, два из них написал я, и они уже изрядно устарели. Могу специально для тебя написать новый. Хочешь? Алсо >>124265 прочитай.
>> No.153937 Reply
File: 1470355904-8baa0e1bf778b8b92d82b1ecf1f5bac6.jpeg
Jpeg, 475.09 KB, 2048×1365 - Click the image to expand
edit Find source with google Find source with iqdb
1470355904-8baa0e1bf778b8b92d82b1ecf1f5bac6.jpeg
>> No.153961 Reply
Анимэхооп, ты тут?
>> No.153966 Reply
File: перекат.jpg
Jpg, 37.69 KB, 343×604 - Click the image to expand
edit Find source with google Find source with iqdb
перекат.jpg
>>153961
А то.
>> No.153967 Reply
>>153966
Так тут никого нет и скорость два поста в месяц. Хотел тебя спросить, а ты кто сам, матёшник? Студент/аспирант? Или кандидат/phd?
>> No.153968 Reply
File: 1426720312709.png
Png, 246.77 KB, 736×732 - Click the image to expand
edit Find source with google Find source with iqdb
1426720312709.png
>>153967
Со временем народу здесь станет больше. Студентота же.
>> No.153970 Reply
>>153968
Не пойму: это цермело или франкель?
>> No.153971 Reply
>>153970
Какая разница? Они слились в вечности.
>> No.154120 Reply
Анон, как доказать, что пересечение всех замкнутых надмножеств M действительно даёт замыкание M?
>> No.154122 Reply
>>154120
Рассмотрим топологическое пространство.
Пусть M - какое-то множество его точек.
Пусть x - такая точка, что любая её окрестность пересекается с M, тогда x называется точкой прикосновения M.
Пусть M' - множество всех точек прикосновения M.
Пусть M'' - пересечение всех замкнутых множеств, содержащих M.
Докажем, что M' = M''.

M'' замкнуто, так как пересечение любого семейства замкнутых множеств замкнуто. Значит, дополнение W множества M'' открыто. Множество открыто тогда и только тогда, когда содержит каждую точку вместе с некоторой её окрестностью. Значит, W содержит любую точку вместе с её окрестностью. Пусть x - точка прикосновения M. Если точка x не принадлежит M'', то она принадлежит W, следовательно, хотя бы одна окрестность точки x не пересекается с M, что невозможно. Значит, x принадлежит M''. Значит, M' подмножество M''.

Пусть x - точка M''. Пусть U - её окрестность. Так как U открыто, дополнение U замкнуто, обозначим его V. Предположим, что U не пересекается с M. Тогда M является подмножеством V, следовательно, существует замкнутое множество, которое является надмножеством M, но элементом которого не является x. Это невозможно, так как x принадлежит всем замкнутым надмножествам M. Значит, предположение неверно. То есть любая точка M'' является точкой прикосновения. Значит, M'' подмножество M'.
>> No.154123 Reply
Посоветуйте хорошую книжку по статистике, например.
>> No.154124 Reply
>>154123
Ван дер Вардена читал?
>> No.154160 Reply
Осенние курсы НМУ появились. В этом году Шарыгин будет читать K-теорию.
http://ium.mccme.ru/f16/f16.html
>> No.154161 Reply
>>154160
K-теория, это хорошо.
мимо К-ракодил
>> No.154183 Reply
File: tarsier-monkey-animal-primate-eyes-cute-2560x1440.jpg
Jpg, 456.12 KB, 2560×1440 - Click the image to expand
edit Find source with google Find source with iqdb
tarsier-monkey-animal-primate-eyes-cute-2560x1440.jpg
Есть ли смысл ходить в НМУ, если ты примат?
Есть ли смысл ходить не на все курсы?
Сколько времени примерно занимает домашка?
>> No.154230 Reply
>>154183
Смысл ходить в НМУ есть всегда. Курсы в НМУ ты выбираешь сам, за исключением обязательных. Домашка - как получится.
>> No.154391 Reply
Перекачусь к вам.
>> No.154417 Reply
File: vWz9KR7sqBY.jpg
Jpg, 135.47 KB, 1280×719 - Click the image to expand
edit Find source with google Find source with iqdb
vWz9KR7sqBY.jpg
Пожалуйста, помогите. Я не уверена, что сделала верно. нужно просто от оси х отложить вниз 1 см, провести горизонтальную линию и отметить на ней точки? или тут что-то другое?
>> No.154418 Reply
File: LR1JI-rhBws.jpg
Jpg, 118.96 KB, 1280×719 - Click the image to expand
edit Find source with google Find source with iqdb
LR1JI-rhBws.jpg
Что делать дальше?
Через какую из пяти точек А нужно проводить и почему? Как отмерить под углом к Плоскости 1?
>> No.154423 Reply
>>154417
>>154418
Лучше в /b/ спроси. Тут же не кафедра черчения всё-таки.
>> No.154427 Reply
>>154423
Извините, я первый раз зашла, поэтому сразу не сориентировалась.
>> No.154437 Reply
File: S-B-T.png
Png, 24.56 KB, 590×225 - Click the image to expand
edit Find source with google Find source with iqdb
S-B-T.png
Не могу понять доказательство теоремы Шрёдера-Бернштейна про антисимметрию кардинальных чисел, в частности ту, что приводится в учебнике. До функции g всё ясно, а дальше ясно чуть ментьше:

- Как проверить биективность g?
- Есть ли в подобных случаях какие-то алгоритмы построения функций или это креативная часть доказательства?
>> No.154439 Reply
File: pict.jpg
Jpg, 49.65 KB, 586×167 - Click the image to expand
edit Find source with google Find source with iqdb
pict.jpg
почему q нечетное, не могу разобрать
>> No.154440 Reply
File: кбш1.PNG
Png, 225.20 KB, 944×826
edit Find source with google Find source with iqdb
кбш1.PNG
File: кбш2.PNG
Png, 182.41 KB, 816×906
edit Find source with google Find source with iqdb
кбш2.PNG

>>154437
По сути, речь идёт вот о чём.
http://www.unn.ru/math/no/9/_nom9_004_begunts.pdf
> это креативная часть доказательства
this
>> No.154441 Reply
>>154439
Если q - четное, то, так как p - чётное, дробь p/q сократима. Но она по условию несократима.
>> No.154442 Reply
>>154441
Блин, как не догадался.. Спасиб, анон
>> No.154444 Reply
Почему у меня пикчерсы не отображаются?
>> No.154446 Reply
>>154444
Спроси в /d/.
>> No.154493 Reply
File: fds.jpg
Jpg, 22.01 KB, 417×161 - Click the image to expand
edit Find source with google Find source with iqdb
fds.jpg
издевался над дробью и так и сяк, но так и не понял этого преобразования. помогите
>> No.154494 Reply
>>154493
Тут не преобразование. Смотри, xn равно твоей дроби.
Дальше ты не преобразовываешь эту дробь, а рассматриваешь другую дробь.
А именно, ты рассматриваешь дробь "xn-1/3", т.е. разность xn и 1/3.
Если посчитать xn-1/3, то как раз всё и получится.
>> No.154495 Reply
File: dsa.jpg
Jpg, 6.43 KB, 417×44 - Click the image to expand
edit Find source with google Find source with iqdb
dsa.jpg
>>154494
Сошлось!
>> No.154496 Reply
>>154495
Так-то.
>> No.154502 Reply
File: fdsfds.jpg
Jpg, 32.67 KB, 608×230 - Click the image to expand
edit Find source with google Find source with iqdb
fdsfds.jpg
>>151715
Матан очень плохо идет, может быть со мной что-то не так? Грызу ночью и днем, но прогресса почти не видно, может перейти на деградирование?
>> No.154504 Reply
>>154502
Вопрос в чём?
>> No.154505 Reply
>>154504
Как получились эти неравенства?
>> No.154506 Reply
>>154505
Пусть a и b положительны.
Если a<b, то 1/b < 1/a, и обратно.

3n^2 + 2n - 4 > 3n^2 - 4 (в левой части на одно положительное слагаемое больше).
3n^2 - 4 > 2n^2 (потому что n>2).
Дальше просто n в числителе и знаменателе сокращаем.
>> No.154507 Reply
>>154506
то бишь, они не выводились друг из друга, а просто очередные предположения, для того, чтобы в итоге убедиться, что разница между пределом и дробью бесконечно малое?
>> No.154508 Reply
File: gfdg.png
Png, 2.45 KB, 203×48 - Click the image to expand
edit Find source with google Find source with iqdb
gfdg.png
>>154507
или вот например, нихуя не могу понять, ну как получилось (a-1)/n?
>> No.154519 Reply
>>154507
Да, они не выводились друг из друга. Это последовательное ослабление предположений. "Если x меньше чем A, то и подавно x меньше чем B, так как A очевидно меньше B". Числа A и B не выводятся друг из друга тривиальными выкладками - имеет место трюк.

>>154508
Оно не "получалось", просто автору очевидно (или он ранее доказал), что "n-ый корень из a минус 1" меньше чем "(a-1)/n".
>> No.154557 Reply
>>154508
Перенеси 1 вправо, возведи в n и используй бином ньютона.
>> No.154585 Reply
Ну что, теперь тут убежище. Может маттред на форчоне создадим?
>> No.154588 Reply
>>154585
Может пора информировать население об окончательном перекате?
>> No.154600 Reply
>>154585
Создайте в /sci/ и киньте ссылки сюда, на брче и в /int/ форчана.
>> No.154602 Reply
>>154585
Нет, здесь уютнее. На форчане всё будет быстро смывать.
>> No.154607 Reply
File: винберг.png
Png, 9.48 KB, 1369×130 - Click the image to expand
edit Find source with google Find source with iqdb
винберг.png
Кстати, для почина задачка.
>> No.154612 Reply
Где списки литиратуры? Где список мемав? Где?! Нук быстро все суда!
>> No.154613 Reply
>>154612
> Тред обучения математике: >>124265
>> No.154614 Reply
Алсо ссылки на сосачевские треды. Пусть будут здесь, мало ли.

Маттреды.
1. https://arhivach.org/thread/18638/
2. https://arhivach.org/thread/27246/
3. https://arhivach.org/thread/27696/
4. https://arhivach.org/thread/38709/
5. https://arhivach.org/thread/46502/
6. https://arhivach.org/thread/48852/
7. https://arhivach.org/thread/52165/
8. https://arhivach.org/thread/56479/
9. https://arhivach.org/thread/63306/
10. https://arhivach.org/thread/70618/
11. https://arhivach.org/thread/74342/
12. https://arhivach.org/thread/74341/
13v1. https://arhivach.org/thread/76561/
13v2. https://arhivach.org/thread/92428/
14. https://arhivach.org/thread/78408/
15. https://arhivach.org/thread/79152/
16. https://arhivach.org/thread/82499/
17. https://arhivach.org/thread/92427/
18. https://arhivach.org/thread/84722/
19. https://arhivach.org/thread/87923/
20. https://arhivach.org/thread/91329/
21. http://arhivach.org/thread/93067/
22. https://arhivach.org/thread/94240/
23. https://arhivach.org/thread/95680/
24. https://arhivach.org/thread/96720/
25. https://arhivach.org/thread/99481/
26. https://arhivach.org/thread/100880/
27. https://arhivach.org/thread/101335/
28. http://arhivach.org/thread/106743/
29. https://arhivach.org/thread/109198/
30. https://arhivach.org/thread/114111/
31. https://arhivach.org/thread/116099/
32. https://arhivach.org/thread/118093/
33v1. https://arhivach.org/thread/122613/
33v2. https://arhivach.org/thread/122615/
34. https://arhivach.org/thread/123717/
35. https://arhivach.org/thread/128822/
36. https://arhivach.org/thread/129039/
37. https://arhivach.org/thread/131462/
38. https://arhivach.org/thread/138362/
39. https://arhivach.org/thread/138429
40. http://arhivach.org/thread/140404/
41. https://arhivach.org/thread/142386/
42. https://arhivach.org/thread/145879/
43. https://arhivach.org/thread/146833/
44. https://arhivach.org/thread/152600/
45. https://arhivach.org/thread/153157/
46. https://arhivach.org/thread/156244/
47. https://arhivach.org/thread/159628/
48. https://arhivach.org/thread/165872/
49. https://arhivach.org/thread/183576/
50. https://arhivach.org/thread/187447/
51. https://arhivach.org/thread/188358/
52. https://arhivach.org/thread/197703/

Для начинающих.
http://arhivach.org/thread/157681/ Математика для начинающих, тред #1
http://arhivach.org/thread/157894/ МАТЕМАТИКА ДЛЯ НАЧИНАЮЩИХ, ТРЕД 2, ТЕПЕРЬ БАНАНОВЫЙ
http://arhivach.org/thread/165665/ МАТЕМАТИКА ДЛЯ НАЧИНАЮЩИХ, ТРЕД 3, ЛАМПОВЫЙ
http://arhivach.org/thread/175432/ МАТЕМАТИКА ДЛЯ НАЧИНАЮЩИХ, ТРЕД 4, ЕЩЕ БОЛЬШЕ КАРТОФАНА
http://arhivach.org/thread/175434/ МАТЕМАТИКА ДЛЯ НАЧИНАЮЩИХ, ТРЕД 5, СЛЕДУЮЩИЙ ЗА ЧЕТВЕРТЫМ
http://arhivach.org/thread/174050/ МАТЕМАТИКА ДЛЯ НАЧИНАЮЩИХ, ТРЕД 6, ПОМОЩЬ НОВИЧКАМ
http://arhivach.org/thread/181477/ МАТЕМАТИКА ДЛЯ НАЧИНАЮЩИХ, ТРЕД 7, ПЕРЕКАТЫ, РЕГЕНЕРАЦИЯ И АНАЛИЗ НА МНОГООБРАЗИЯХ
http://arhivach.org/thread/177783/ МАТЕМАТИКА ДЛЯ НАЧИНАЮЩИХ, ТРЕД 8, ИДУЩИЙ ПЕРЕД ДЕВЯТЫМ
http://arhivach.org/thread/180129/ МАТЕМАТИКА ДЛЯ НАЧИНАЮЩИХ, ТРЕД 9, ЮБИЛЕЙНЫЙ!
http://arhivach.org/thread/188615/ МАТЕМАТИКА ДЛЯ НАЧИНАЮЩИХ, ТРЕД 11, N-ПЕТУХ НАНОСИТ ОТВЕТНЫЙ УДАР!
http://arhivach.org/thread/190551/ МАТЕМАТИКА ДЛЯ НАЧИНАЮЩИХ, ТРЕД 12, ОТПУСКНОЙ!
https://arhivach.org/thread/195174/ МАТЕМАТИКА ДЛЯ НАЧИНАЮЩИХ, ТРЕД 13, СКОРО В ШКОЛУ!
https://arhivach.org/thread/194110/ МАТЕМАТИКА ДЛЯ НАЧИНАЮЩИХ, ТРЕД 14 : [Запрещенного переката тред]
https://arhivach.org/thread/194111/ МАТЕМАТИКА ДЛЯ НАЧИНАЮЩИХ, ТРЕД 15 : Познаём Секреты
https://arhivach.org/thread/195175/ МАТЕМАТИКА ДЛЯ НАЧИНАЮЩИХ, ТРЕД 16 : Решаем задачи
https://arhivach.org/thread/196701/ МАТЕМАТИКА ДЛЯ НАЧИНАЮЩИХ, ТРЕД 17 : Имени Понтрягина Льва Семёновича
https://arhivach.org/thread/198979/ МАТЕМАТИКА ДЛЯ НАЧИНАЮЩИХ, ТРЕД 18 : 5 Гротендиков в минуту
https://arhivach.org/thread/200701/ МАТЕМАТИКА ДЛЯ НАЧИНАЮЩИХ, ТРЕД 19 : Продолжаем сидеть в клетке
https://arhivach.org/thread/202249/ МАТЕМАТИКА ДЛЯ НАЧИНАЮЩИХ, ТРЕД 20: Комбинаторики едишон
>> No.154617 Reply
>>154613
Там нет ничиво, ты вриош!
>> No.154618 Reply
>>154617
В треде-то? В треде есть.
>> No.154619 Reply
Шалом селяне. Давайте вы мне поможете применять математику для реальной жизни, без регистрации и смс.
Пределы, лифференциалы, интригалы, эволюты и теорему гаусса остроградского знаю, но применить их на реальное дело не могу.
Вот например, мне надо рассчитать гидравлическое сопротивление для трубы для водорода под давлением пять атмосфер. Посчитать все это по бойлю мариоту я могу, но у меня получается что это сопротивление КРАЙНЕ МАЛО и чуть ли не отрицательно.
>> No.154621 Reply
Ничего там не будет смывать, одно время даже аналогичные нашим серии тредов были, где толковые форчеры поясняли за топосы и трап-алгебраический-геометр пояснял за свою тяжёлую жизнь.
>> No.154624 Reply
>>154621
> трап-алгебраический-геометр
Пруфы были?
>> No.154625 Reply
>>154614
Как завсегдатай тредов математики обоих направлений могу сказать что они выродились в мемчики, бурбакизм, выражением единицы церез теорию множеств и постоянные репорты на тех кто хочет прекратить этот балаган.
Другими словами, они тут не нужны.
борода сказано
>> No.154628 Reply
Аноны, поясните за дифференциал композиции. Как доказать, что o-малое в случае композиции - это действительно o-малое?
>> No.154631 Reply
>>154628
Бесконечно малые/большие знаешь? Или только калечные пределы?
хозяйству бережет
>> No.154636 Reply
>>154628
О каком дифференциале речь?
>> No.154639 Reply
Посоветуйте материал по отражениям в теории множеств. На лекции вообще ничего не понял.
>> No.154640 Reply
>>154639
По отражениям или по отображениям?
>> No.154642 Reply
>>154636
Фреше.
>> No.154668 Reply
>>154640
Отображениям.
>> No.154706 Reply
File: uravneniya.png
Png, 3.46 KB, 174×174 - Click the image to expand
edit Find source with google Find source with iqdb
uravneniya.png
Доброаноны, помогите первокурснику провинциальной шаражки научиться решать подобные системы уравнений с множествами. Посоветуйте теории по этой теме, ибо сам найти не могу.
>> No.154713 Reply
>>154706
Распиши через предикаты, упрощай с помощью свойств множеств (транзитивности всякие) и свойств логики.
>> No.154714 Reply
>>154713
Какие предикаты? Свойств мы не учили таких.
>> No.154718 Reply
>>154502
Доброчую
>> No.154723 Reply
>>154706
Тут можно порассуждать.
Если два множества пересечь, то объединение с любым из них будет давать это множество.
А пересечь с Б объеденить с Б = Б
А пересечь с Б объеденить с А = А
Это касается и обратно, т.е. пересечения:
А объединить с Б перечь с Б = Б
А объединить с Б перечь с А = А
И этим можно пользоваться, чтобы упрощать равенства. Мы ведь можем с левой и правой частью делать одно и то же (если это действие определено и допустимо).
Так же можно брать разность левой части от правой или правой от левой. Т.к. они равны, то их разность равна пустому множеству.
>> No.154724 Reply
>>154706
А об. Х = Б пер. Х \\ объединим с Х
(А об. Х) об. Х = (Б пер. Х) об. Х \\ (объединение ассоциативно, как и пересечение, коммутативно, а объединение и пересечение с самим собой даёт самого себя; а вот взаимодействуют объединение и пересечение по законам дистрибутивности; но из соображений озвученных выше - справа у нас получиться Х - отчего? Потому что если некое множество А есть подмножество Б, то объединение А и Б - есть Б; притом надо иметь в виду, что пересечение двух множеств всегда есть подмножество каждого из них.)
А об. Х = Х \\ это, в частности, значит, что А подмножество Х, может быть собственное. И отсюда, внезапно, следует, что пересечение А и Х есть А.
Впрочем, это может стать ясным, как только мы посмотрим на первое уравнение.
Тогда второе примет вид:
А = С об. Х
Слева у нас подмножества Х, следовательно объединение справа не может быть больше X, но оно не может быть и меньше Х! (по определению), тогда справа у нас Х и получаем, что:
А = Х
>> No.154725 Reply
>>154724
Хотя тут и проще можно было рассудить. Справа в первом уравнении подмножество Х, значит слева так-же подмножество Х, а значит А подмножество Х. Ведь объединение содержит в качестве подмножеств свои слагаемые.

А \ Х = Х \ Б
Х \ А = С \ А
Из последнего не следует, что Х = С. Ведь разность множества не тождественна вычитанию или делению!
Если мы берём разность Х и какого-то множества, то результат будет подмножеством Х. Т.е. справа в первом уравнении у нас подмножество Х, следовательно, и слева у нас подмножество Х. Но! Слева из множества А мы забираем все элементы содержащиеся в Х! Что же остаётся? Пустое множество. Т.е. А есть подмножество Х. А вот теперь очевидно, что Х = С, т.к. и те элементы из Х которые мы отняли от С содержатся в Х и те, что остались (что следует из второго уравнения).
>> No.154726 Reply
>>154725
Т.е. чтобы из последнего вывести, что X = С, надо знать, что А - подмножество Х, а это видно из первого. Приведу пример:
Х = {1, 2, 3}, C = {1, 2, 4}, A = {3, 4}
Тут Х\А = С\А, но, как видно, Х =\= С.
Хотя, кажется, выше я напиздел. Ведь может быть и так:
X = {1, 2, 3, 4}, C = {1, 2, 4}, А = {3, 4}
Тогда Х\А = С\А, но, как видно, Х =\= С. Притом, что А подмножество X.
Из последнего можно вывести, выходит, что С - есть подмножество Х.

Подумаю ещё...
>> No.154729 Reply
>>154726
Так, подумалось мне, что и в первой я допустил какую-нибудь ошибку, ведь какого члена результат зависит только от множества А?

Допустим: А = {1, 2, 3, 4}, C = {1, 5}, B = {2, 3}.

{1, 2, 3, 4} об. Х = {2, 3} пер. Х \\ Собственно Х =\= А уже на первом шаге. И более того, это не уравнение не имеет решений. Так что второе даже смысла нет рассматривать.
Т.е. всё рассуждение выше было построено на имплицитной посылке, что уравнения имеют решения и, следовательно, налагало ограничения и на С, B.

Как это в общем виде решать? Да хуй его знает. А ещё и так, чтобы правильно!
>> No.154730 Reply
>>154729
> не уравнение
уравнение

Попробую ещё раз.

В левой части первого уравнения у нас объединение с Х, а в правой - пересечение. По определению: Х пересечь с Y =< X объединить с Y (если они совпадают, то равно, если отличаются, то в их пересечении будет меньше элементов, чем в объединении).
Тут, на самом деле, работает ещё более сильное отношение: X пер Y =< X об Z, причём не важно, как между собой соотносятся множества Y, Z и Х. Почему? Да просто потому, что объединение всегда больше и равно самого множества (как объединения с пустым множеством), а пересечение всегда меньше или равно самого множества (как пересечение с самим собой).

Левая часть содержит все элементы А и все элементы Х - и она в точности больше или равна и Х, и А.
В левой, как пересечении Х с Б, следовательно, должны содержаться все элементы и из Х, и из А. А возможно это только тогда, когда Х содержит А в качестве подмножества, а Б содержит в качестве подмножества и Х, и А.

Т.е. мы знаем, что А подмн. Х подмн. Б.

Из этого следует, что пересечение А и Х есть А. Тогда во втором уравнении получается, что:
А равно объединению С и Х. Если С содержит хотя один элемент не принадлежащий А, то уравнение не будет иметь решений! Потому должно С подмн. А. Это относится и к Х. Откуда следует, что Х подмн. А. (Если некое множество есть объединение двух других, то эти два - есть его подмножества.)
Из того, что: А подмн. Х и Х подмн. А следует, что А = Х.

Вот. Но решить такое уравнение можно, только если выполняется, что: С подмн. А подмн. Б
>> No.154731 Reply
>>154730
Попытаюсь так же рассудить для второго.
Разность А и Х подмн. А, это понятно. Разность Х и Б подмн. Х. Если левая часть подмножество Х, то и правая, и наоборот.
Важно и то, что левая часть не содержит тех элементов Х, которые содержаться в А. Но т.к. левая часть является при этом подмножеством Х, то она, внимание, пуста. Откуда видно, что А подмножество Х.
Из второго уравнения видим, что Х содержит С. Откуда? От противного. Правая часть есть подмножество Х, как разность Х и А. Пусть есть хотя бы один элемент с из С, который не принадлежит Х, тогда разности С\А - будет так же содержать этот элемент (ведь он не входит в Х, а значит и в его подмножество А), следовательно, правая часть уравнения не будет равняться левой (!).

Вообще, Х = (X\A) об. A, т.к. А подмн. Х.
Тогда получаем, что Х = (С\А) об. А или С об. А, т.к. С содержит все элементы С\А и не содержит таких элементов, которые бы не содержались в А или Х.

Подставим в первое.
А\(С об. А) = (С об. А)\B
A\C ob A\A = C\B ob A\B
A\C = C\B ob A\B

Вот тут, честно, затупил. Условие не слишком изящное, так что думаю, что где-то ошибка. Или его можно сформулировать иначе.

В общем, ребята, отчего не помогаете? Студент, ты зачем спрашивал?
>> No.154736 Reply
- Как представить сдвиг на вектор лежащий на, вращение и трансформацию схожести плоскости на языке теории множеств?

- Что такое взаимная однозначность функций?
>> No.154741 Reply
File: VMBSku2mfIc.jpg
Jpg, 496.36 KB, 2304×1728 - Click the image to expand
edit Find source with google Find source with iqdb
VMBSku2mfIc.jpg
Анон, поступил в универ, начался матан. Пикрил.
У xn есть предел, доказал по Коши. У минус одной третьей в энной степени предел тоже, очевидно есть. Значит можно записать все так, как я записал. Но это в то же время и неправильно совсем. В чем я ошибся и почему?
>> No.154742 Reply
>>154741
В том, что надо было представить это в виде мат. индукции.
Для n доказано существование предела. Покажем, что и для n + 1 есть передел. Ну и вот.

А ошибся в чём? Ну, тут вообще не заметно, что это доказательство. Где все слова, кванторы, где использовано определение или теорема, заключение? Где ты тут критерий Коши применил? Где?
Как известно даже мне, стремление к нулю свободного члена НЕОБХОДИМЫЙ, но не ДОСТАТОЧНЫЙ признак.
>> No.154746 Reply
>>154742
Ах да, про мат. индукцию это шутка такая это не шутка, просто об этих темах давно не рассуждал и тупанул, тут же у нас предел, а на предельные случае вывод об n+1 может не распространяться; ну, S0 = 1, S1 = 1 + 2 = S1 + S2, ..., S{n+1} = Sn + (n + 1); да, S0 базис или посылка индукции, индукционное предположение, что Sn - конечное; а переход, что тогда конечно и S_{n + 1} - но в переходе предельном это нихуяшеньки не так, т.к. последовательность расходящаяся!

Короче, доказывай по критерию Коши и не выёбывайся! А вот это шутка. Ну, я про грубый тон. А критерий Коши тут действительно к месту, надо оценить остаток по соответствующей формуле, посмотри. Хотя там и другие методы были... надо вспомнить. Интегральный критерий Коши был, потом, Лагранджа, с отношением и Даламбера? Ну, в учебники должно быть написано.

Т.е. необходимый признак выполняется, а вот стремление самой последовательности к нулю ты не доказал. Как я тебя понял, ты рассматриваешь последовательность сумм, чтобы оценить сходимость ряда. Ну, понятно, если последовательность эта стремиться к нулю, то ряд сходится. Вот только ты не показал, что сумма стремиться к нулю. Я про x_n, да, к нему прибавляется убывающая величина, но и 1/n убывающая величина, только если мы начнём её куда-нибудь собачить, ничего не сойдётся.

А тут ряд знакопеременный, кстати, может для него достаточно стремления к нулю |xn - x{n+1}| ? Сейчас погуглю... Ох, Лейбница. Да, ты применил признак Лейбница.

Короч, я сейчас всю тему выучу, если так дальше дело пойдёт.
Просто проверь два условия:
1) xn < x{n+1} (Sn - сумма, xn - её члены, просто у тебя странное обозначение).
2) \lim{x \to +\infty} xn \to 0
Ну и напиши, у нас тут, бла-бла-бла, знакопеременный ряд, применим критерий Лейбница.
Мы применили критерий Лейбница, ряд сходиться. Причём сходится он и абсолютно, как геом. прогрессия.
>> No.154757 Reply
File: 2016-09-30_19-47-47.jpg
Jpg, 122.42 KB, 875×568 - Click the image to expand
edit Find source with google Find source with iqdb
2016-09-30_19-47-47.jpg
Что могут означать эти (-1) (-2) на пикрилейтед и к чему они вообще относятся?
>> No.154758 Reply
>>154757
Логарфмические индексы.
>> No.154759 Reply
>>154758
Можешь пояснить, как пересчитать на простое число с плавающей запятой?
>> No.154760 Reply
>>154759
А все, всосал, спасибо.
>> No.154772 Reply
>>154736
Anybody?
>> No.154782 Reply
>>154736
> - Как представить сдвиг на вектор лежащий на, вращение и трансформацию схожести плоскости на языке теории множеств?
Вопрос не очень понятен. Уточни пожалуйста, что конкретно ты хочешь и зачем.

> - Что такое взаимная однозначность функций?
Что такое взаимно однозначное отображение? В каждой точке есть единственное значение, а для каждого значения есть единственная точка, в которой это значение достигается.
>> No.154787 Reply
>>154782
Мне надо доказать, что множество плоскости стабильно (или какая-то хуйня, не важно) относительно сдвига на вектор лежащий на ней, вращения вокруг точки на ней, трансформации схожести или любой их композиции. А я с этой книжки начал математику, я не знаю всей этой геометрической поебени и не слишком знать хочу сейчас.
> Что такое взаимно однозначное отображение?
Нет, что такое взаимная однозначность двух функций?
>> No.154791 Reply
>>154642
Лоран Шварц, глава про производную. Там есть целый параграф про это.

>>154668
Любая книжка по теории множеств. Например Архангельский, "Канторовская теория множеств".

>>154736
https://en.wikipedia.org/wiki/Shift_operator
https://en.wikipedia.org/wiki/Rotation_(mathematics)
https://ru.wikipedia.org/wiki/Изометрия_(математика)
https://ru.wikipedia.org/wiki/Биекция

>>154787
Почему бы тебе не пропустить это задание и не читать книгу дальше? Не надо делать всё подряд как трактор, это же учебник, а не повеление.
>> No.154792 Reply
>>154791
Началось. Я не за советами по жизни пришел, а по конкретным вопросам.
>> No.154793 Reply
>>154792
Скорее всего, ты некорректно сформулировал вопросы. Покажи скрин из книги. Что в ней написано на самом деле?
>> No.154800 Reply
>>154787
> Мне надо доказать, что множество плоскости стабильно (или какая-то хуйня, не важно) относительно сдвига на вектор лежащий на ней, вращения вокруг точки на ней, трансформации схожести или любой их композиции.
> я не знаю всей этой геометрической поебени и не слишком знать хочу сейчас.
Тебе нужно доказать, что евклидова плоскость переходит сама в себя при этих преобразованиях, так? Чтобы представить эти операции на языке, нужно сначала понять, а что же это за действия. Может всё-таки стоит с геометрии начать? Без понимания это всё бессмысленно.

> я с этой книжки начал математику
Покажи-ка свою книжку.

>>154792
Ну и зря. Тот анон тебе годный совет дал.
>> No.154803 Reply
>>154800
Надо доказать, что плоскость будет иметь неподвижную точку (она равна своей функции) при любой композиции сдвига, вращения и гомотетии с коэффициентом меньше 1.
> Чтобы представить эти операции на языке, нужно сначала понять, а что же это за действия. Может всё-таки стоит с геометрии начать? Без понимания это всё бессмысленно.
Настоящее фундаментальное понимание дает теория множеств. Ты же не думаешь, что я не могу представить вращение плоскости вокруг своей точки?

Я и не стою на месте, застряв на задаче - мне просто не нравится, когда спрашиваешь как подавать стейк, а тебе советуют на вегетерианство перейти. Все постоянно это делают. Низшая форма общения и оправдывает выражение "advice is the worst vice".

Отвечай по существу. Нечего сказать? Молчи.
>> No.154807 Reply
>>154803
> Ты же не думаешь, что я не могу представить вращение плоскости вокруг своей точки?
> я не знаю всей этой геометрической поебени и не слишком знать хочу сейчас.
Does not compute.
>> No.154808 Reply
>>154807
Я все еще не знаю что есть гомотетия по теории множеств. То есть если мы возьмем, например, треугольник в 2д и гомотетируем его, их центры будут совпадать? Я так и не получил непосредственного определения гомотетии и соответственно понятия не имею, что она сделает с плоскостью. На вики хуйня какая-то, на других сайтах тоже.

Если гомотетия это просто увеличение/уменьшение с сохранением некоего центра, как я это понял, то почему гомотетия плоскости с коэффициентом больше или равным единице не сохранит неподвижную точку? Ведь гомотетия плоскости это тождественная функция?
>> No.154810 Reply
>>154803
А это, случаем, не из Куранта-Робинса?
>> No.154811 Reply
>> No.154813 Reply
>>154808
> Если гомотетия это просто увеличение/уменьшение с сохранением некоего центра, как я это понял, то почему гомотетия плоскости с коэффициентом больше или равным единице не сохранит неподвижную точку? Ведь гомотетия плоскости это тождественная функция?
Сохранит. Из формулировки задания этого не следует. Т.е., эм, если даже мы ограничиваемся рассмотрением какого-то частного случая, это ещё не значит, что выводы на него распространяемые должны с обратным знаком применяться к остальным. И это легко проверить, взяв гомотетию с коэффициентом 1, которая, эм, ничего не делает, просто сохраняя плоскость такой, какая она есть.
>> No.154814 Reply
File: shift,-rotation-and-simtrans.png
Png, 12.64 KB, 575×54 - Click the image to expand
edit Find source with google Find source with iqdb
shift,-rotation-and-simtrans.png
>>154811
> я не знаю всей этой геометрической поебени и не слишком знать хочу сейчас.

Приведу пример.

Надо доказать, что при сдвиге плоскости на лежащий на ней вектор, она будет иметь устойчивое подмножество. Такая функция будет ф(х) = (х + в). Представляя данное множество квадратом действительных чисел, мы видим, что эта функция тождественна. Соответственно, очевидно она имеет устойчивое подмножество.

Надо доказать, что при вращении плоскости относительно лежащей на ней точки (x0), множество будет иметь инвариантное подмножество. Пусть данная плоскость X C R^3. Функцию можно определить как f(X) = {y in Y C R^3 | for every x c X, x != x0 exists v [x + v = y /\ x0 = y0 /\ for every x1, x2 (y0y1 || y0y2)]}. Понятно, что инвариантой будет {x0}.

Но это, кстати, не совсем те задачи, что в учебнике. В учебнике просто просят описать множества, которые будут устойчивы сдвигу и инвариантны вращению плоскости. Очевидно, это метрические пространства, содержащие вектор сдвига и сферы с точкой в центре соответственно.

А вот собственно та самая задача. Мне понятно, как композиции сдвига и вращения сохранят неподвижную точку, но я просто не знаю как описать превращение схожести плоскости.

>>154813
Ну это я уже человеческий фактор применил и спросил ради подтверждения изначального понимания.
>> No.154815 Reply
Анон, интересует ситуация в топологии. Мне тут сказали, что общая топология умирает и нет смысла ей заниматься или даже углублённо изучать. Что думаете?
>> No.154816 Reply
>>154815
Математика вообще умирает, её заменяет нейронаука очень и очень быстро.
>> No.154817 Reply
>>154815
Пошли их куда подальше и занимайся тем, к чему лежит душа.
>>154816
> математика
> заменяет нейронаука
В голос.
>> No.154819 Reply
>>154817
Это понятно, но всё же интересно, что сейчас с общей топологией. Мне сказали, что сегодня ей занимается очень мало людей (чуть ли не единицы). Меня это удивило. Казалось, что она более популярна.
>> No.154820 Reply
>>154819
Какая тебе разница, блядь? Ты еще очень далёк от того, чтобы совать нос в такие вещи. Просто сиди и учи сейчас. Будешь тише воды, ниже травы - авось позицию teaching assistant получишь. Главное не высовываться слишком и не интересоваться "политикой". Как есть - так и надо.
>> No.154822 Reply
Мне так и не ответили что такое взаимная однозначность двух функций?
>> No.154823 Reply
>>154822
Это из-за того, что ты неблагодарная свинья.
>> No.154824 Reply
>>154823
Ну не бугурти, благодарен я, благодарен.
>> No.154825 Reply
>>154824
Может имеются в виду взаимно обратные функции? Там нет определения этого замысловатого отношения?
>> No.154826 Reply
>>154825
А. Я неправильно понял текст, лол. Никогда не мыслите.
>> No.154853 Reply
File: c-to-c2.png
Png, 2.48 KB, 291×53 - Click the image to expand
edit Find source with google Find source with iqdb
c-to-c2.png
Я правильно понял, что это будет биекцией только при бесконечном количестве С?
>> No.154861 Reply
>> No.154862 Reply
>>154853
И при четном же.
>> No.154863 Reply
Эх блядь, так близко к самостоятельному доказательству теоремы Шредера-Бермунда был - рисунок хуёвый нарисовал. Если бы только правильный рисунок сделал, всё стало бы ясно.
>> No.154864 Reply
>>154862
Уверен? Проверь для n = 3.
>> No.154865 Reply
>>154863
Ты хотя бы знаешь что за теорема... здорово же.
>> No.154867 Reply
File: pleb.jpg
Jpg, 139.19 KB, 579×527 - Click the image to expand
edit Find source with google Find source with iqdb
pleb.jpg
>>154864
> > при четном же
> проверь для n = 3
>> No.154877 Reply
>>154867
Жаль, что я не умею читать по английски :(
>> No.154892 Reply
File: 555.jpg
Jpg, 25.94 KB, 600×375 - Click the image to expand
edit Find source with google Find source with iqdb
555.jpg
>>154877
> 2016
> doesn't know english
gg fam
>> No.154900 Reply
>>154892
Спасибо, что не
> 2к16
>> No.154901 Reply
>>154900
бля missed a bit
>> No.154904 Reply
>>154814
Читнул немного геометрии и вот эти три функции в задаче стали ясны. Но при этом мне теперь нихуя не ясно, каким образом у такой композиции будет неподвижная точка? Даже если допустить, что у вращения и превращения подобия общая центральная точка, сдвиг же её сдвинет!
>> No.154936 Reply
Вопрос такой. Вот есть Бурбаки, том по теории множеств, там излагается теория множеств с основ, необходимая для дальнейшего чтения. Но при этом там используется куча устаревших терминов, какой-то гильбертовский формализм, вместо категорий какие-то намекающие структуры. Но вот вопрос, а есть ли что-то подобное, от метаматематики к теории множеств, но при этом, чтобы это не было собственно учебником по метаматематике теории множеств, а учило правильному современному формализму?
>> No.154941 Reply
>>154936
> учило правильному современному формализму
За що?!
>> No.154952 Reply
>>154936
Теория множеств Бурбаки - это на самом деле не метаматематика, а метаметаматематика. Это универсальный фреймворк для обоснования теорий первого порядка. Они вводят теоретико-множественный язык так, чтобы во всех последующих теориях первого порядка можно было с самого начала использовать коллективизирующие предикаты и множества, не боясь порочного круга.
> гильбертовский формализм
> устаревших терминов
Лол.

> правильному современному формализму
Правильный современный формализм в своём языке свободно пользуется наивными множествами, не утруждая себя их обоснованием. Формальная теория множеств в 100% случаев вводится с использованием неформальной теории множеств. Нужна реальная строгость - прочитай Бурбаки. Тем более первый том читается влёт, в нём нет ничего сложного.
>> No.154954 Reply
File: captcha-2.png
Png, 1.36 KB, 300×20 - Click the image to expand
edit Find source with google Find source with iqdb
captcha-2.png
>>154952
Я начал читать их определение единицы, но сбился где-то на триллионном символе.
>> No.154955 Reply
>>154954
Ты лжец же. Они определяют 0 как кардинал пустого множества, единицу как кардинал {0}.
>> No.154974 Reply
File: brouwer.png
Png, 653.19 KB, 1709×744 - Click the image to expand
edit Find source with google Find source with iqdb
brouwer.png
В чем смысл обсуждать в 21 веке бурбаков и т.п., после того как Гедель, а задолго до него Брауэр, доказали принципиальную ограниченность формализма? Я не хочу сказать, что неконструктивная математика совсем не нужна, просто для оснований она не годится, это медицинский факт.
>> No.154975 Reply
File: A7BRncaCEAEHizC.jpg
Jpg, 104.25 KB, 600×522 - Click the image to expand
edit Find source with google Find source with iqdb
A7BRncaCEAEHizC.jpg
>>154955
> Ты лжец же.
>> No.154976 Reply
>>154952
Да у меня просто бомбит от этого формализма. Я не понимаю зачем. Почему именно эпсилон оператор. Чтобы избавиться от аксиомы выбора? Но зачем?
>> No.154980 Reply
>>154976
От аксиомы выбора интуиционистам печёт. Конечно, им от всего печёт, но такова реальность их бытия.
>> No.154981 Reply
File: бурбаки-определение-1-же.png
Png, 64.51 KB, 646×278 - Click the image to expand
edit Find source with google Find source with iqdb
бурбаки-определение-1-же.png
>>154975
> прим. ред.
А ты не примечания показывай, а определения. Определение вот.
>> No.154982 Reply
>>154981
Но я хотел разобраться до конца и начал читать настоящее определение, а не какую-то неформальную сокращенную запись.
>> No.154984 Reply
>>154982
> неформальную
Это не так, использованному символу дано формальное определение. Тау-нотация нужна только в первой главе - чтобы обосновать логику.
>> No.154997 Reply
Кстати, касательно знаменитых упражнений на доказательство тавтологий в теории множество. Как правильно доказать это? Опираясь на аксиому экстенсиональности?
>> No.154999 Reply
>>154997
Да как угодно, лишь бы без лжи. Вообще да, на эту аксиому.
>> No.155003 Reply
> Бурбаки
Кстати, запощу ссылку для слоупоков, а то вдруг не все видели.
http://www.springer.com/fr/book/9783662493601
Видно, Хаим с того света продолжает дело это.
>> No.155004 Reply
>>155003
> с того света
Ты ошибся разделом.
>> No.155005 Reply
>> No.155029 Reply
File: mat.jpg
Jpg, 302.62 KB, 1539×2160 - Click the image to expand
edit Find source with google Find source with iqdb
mat.jpg
Может кто проверить, правильно ли я решаю это дерьмо.
Я, конечно, не самый умный первокурсник, но и нам внятно не объяснили
>> No.155030 Reply
>>155029
Вроде правильно. Никогда не любил задачи на явное нахождение эпсилон-дельта.
>> No.155032 Reply
File: image509.png
Png, 1.63 KB, 275×63 - Click the image to expand
edit Find source with google Find source with iqdb
image509.png
Привет, я тут решил понять, как устроены машина Поста и Тьюринга, залез в теорию автоматов, потом нашел теорему Поста о полноте системы булевых функций, и вообще не понял, как эта теорема связана с машиной Поста. Или они не связаны? С дискретной математикой я знаком на уровне школы, так что почти ничего не понял. Няши, поясните.
>> No.155041 Reply
>>155029
У тебя там даже определение предела неправильное. У тебя СУЩЕСТВУЕТ эпсило такой, что для любого дельта. Начинать надо с того, что для любого эпсилон существует дельта0 такое, что для любого дельта<дельта0 f(дельта)<эпсилон. Фиксируй эпсилон и найди функцию дельта0(эпсилон). Ну и да, решая эту же самую систему уравнений.
>> No.155042 Reply
>>155041
А разве это определение предела?
>> No.155043 Reply
>>155032
Они никак не связаны. Теорема отдельно, машина отдельно. Хотя теорему, при желании, можно доказать с использованием машины, машина не необходима для теоремы.
>> No.155044 Reply
>>155042
https://ru.wikipedia.org/wiki/%D0%9F%D1%80%D0%B5%D0%B4%D0%B5%D0%BB_%D1%84%D1%83%D0%BD%D0%BA%D1%86%D0%B8%D0%B8#.D0.9F.D1.80.D0.B5.D0.B4.D0.B5.D0.BB_.D1.84.D1.83.D0.BD.D0.BA.D1.86.D0.B8.D0.B8_.D0.BF.D0.BE_.D0.9A.D0.BE.D1.88.D0.B8
По Коши — да. Ну там вместо дельта ноль — дельта, а вместо дельты — x-x0. Но суть не в обозначениях, а в кванторах. Ты перепутал «существует» и «для любого». Ну или не ты, а автор того сообщения.
>> No.155046 Reply
>>155043
А как ее доказать на машине? Просто я сейчас тренажер даже для этого скачал, суть ее работы более-менее понял. Сейчас вот загуглил на тему, но ничего не могу найти.
>> No.155058 Reply
Умоляю не отпугнитесь плохим качеством и проверьте мои доказательства. Следующий раз ручкой напишу.
>> No.155059 Reply
File: IMG_0568.JPG
Jpg, 1423.93 KB, 3264×2448
edit Find source with google Find source with iqdb
IMG_0568.JPG
File: IMG_0569.JPG
Jpg, 1348.29 KB, 3264×2448
edit Find source with google Find source with iqdb
IMG_0569.JPG
File: IMG_0570.JPG
Jpg, 1350.25 KB, 3264×2448
edit Find source with google Find source with iqdb
IMG_0570.JPG
File: IMG_0571.JPG
Jpg, 2182.97 KB, 2448×3264
edit Find source with google Find source with iqdb
IMG_0571.JPG

>> No.155060 Reply
>>155059
Это ж поди разбери, что там... хм.
>> No.155063 Reply
File: 9706623.png
Png, 31.75 KB, 904×201
edit Find source with google Find source with iqdb
9706623.png
File: Aleph-script.png
Png, 2.28 KB, 84×142
edit Find source with google Find source with iqdb
Aleph-script.png
File: 5195__93688.14440...
Jpg, 15.72 KB, 300×300
edit Find source with google Find source with iqdb
5195__93688.1444086990.800.800.jpg

>>155059
2. x с плюсиком по определению содержит все элементы x и вдобавок элемент {x}. То есть для любого x с плюсиком существует y такой, что y - элемент x с плюсиком. Поэтому любой x с плюсиком непуст. Не надо тут извращаться.

3. Не понял. Ты в конце утверждаешь, что {{x}} равно x?

4. Чем обосновано утверждение, что если x не является пустым множеством, то x - n-тый последователь пустого множества? Ведь разве не это самое утверждение ты, по сути, сейчас доказываешь? Похоже, что у тебя там порочный круг.

1. А что такое b(x)? Кстати, рукописный алеф пишется вот так.
>> No.155064 Reply
>>155063
3. Я допускаю, что х != у, из чего исходит, что {{х}} = х. Противоречие.

4. Угу, я это уже стёр.

1. Биекция.
>> No.155072 Reply
>>155064
3. Окей, я понял. Прикольное рассуждение.
1. А как у тебя обосновывается, что N не равномощно никакому элементу N?
2. Не заметил сразу. Там написано, что существует такое x, что x не равно пустому множеству и x подмножество пустого множества?
>> No.155075 Reply
File: IMG_0572.JPG
Jpg, 2942.84 KB, 3162×1641
edit Find source with google Find source with iqdb
IMG_0572.JPG
File: IMG_0574.JPG
Jpg, 1341.41 KB, 3256×695
edit Find source with google Find source with iqdb
IMG_0574.JPG
File: IMG_0575.JPG
Jpg, 2145.12 KB, 3264×2448
edit Find source with google Find source with iqdb
IMG_0575.JPG

>>155072
> 1. А как у тебя обосновывается, что N не равномощно никакому элементу N?
Ну по определению алеф нуля он больше любого конечного числа, а любое натуральное число по определению конечное.
> 2. Не заметил сразу. Там написано, что существует такое x, что x не равно пустому множеству и x подмножество пустого множества?
Там по-моему супоз был перед этим. Хуй знает, я переписал уже. Вот по-новому.

И еще одно упражнение в довесок. После этого только ручка. :3
>> No.155089 Reply
>>155075
Осмелюсь совершить акт бампа.
>> No.155173 Reply
>>155089
Это было ошибкой.
>> No.155174 Reply
>>155089
А что ты хочешь услышать, собственно?
>> No.155175 Reply
>>155174
"Все правильно, малаца, вот тебе няша". А вообще - правду, конечно. Правильно написал всё? В т.ч. оформление. Хотя бы задачу с двух правых пиков последнего поста.
>> No.155176 Reply
>>155175
А что ты доказываешь-то на этих пиках?
>> No.155177 Reply
>>155176
На первом что кардинальность последователя произвольного элемента натуральных чисел Фон Нюмана больше нулевого множества.

На второтретьем что для х и y (снова-таки из Неймановских натуральных), хотя бы один из которых является подмножеством другого, их кардинальность либо равна, либо у одного больше, чем у другого. Очевидно, без использования свойств кардинальных чисел.
>> No.155178 Reply
>>155177
Хм. Тогда последний вопрос, кардинальность ты определяешь как что?
>> No.155179 Reply
File: запретить-топологию.jpg
Jpg, 69.62 KB, 685×755 - Click the image to expand
edit Find source with google Find source with iqdb
запретить-топологию.jpg
Как пить дать, гомологией занимаются.
>> No.155180 Reply
>>155178
Как функцию на множествах, значения которой линейно упорядочены.
>> No.155181 Reply
>>155180
Не понял.
>> No.155182 Reply
>>155181
А нет, доказать на правых двух пиксах надо полноту на Неймановских натуральных относительно включения, то есть левое выражение вверху левого пика.

А кардинальность - это множество классов эквиполлентности. О нем известны только транзитивность, антисимметричность и полнота относительно >=, а также то, что булеан всегда высококардинальнее своего аргумента.
>> No.155198 Reply
File: image.jpg
Jpg, 54.24 KB, 400×533 - Click the image to expand
edit Find source with google Find source with iqdb
image.jpg
>>155179
> на пике линал
> запретить топологию
>> No.155207 Reply
>> No.155209 Reply
File: 1456854928001.jpg
Jpg, 67.02 KB, 602×653 - Click the image to expand
edit Find source with google Find source with iqdb
1456854928001.jpg
26 лвл ИТТ. Дело в том, что я где-то после 5-го класса забил на математику, но в силу некоторых причин мне необходимо начать понимать её, поэтому реквестирую хорошую книгу, которая помогла бы мне понять СУТЬ математики, и научиться разбираться в этом. Ну и чтобы там задачки были, всё такое. На русском, ибо я новичок, на английском математику вряд ли пойму, хотя в английской мове я далеко не ньюфаг.
Добра всем, ребята.
>> No.155214 Reply
File: uspenski_apolog_matem_200.gif
Gif, 12.86 KB, 200×311 - Click the image to expand
edit Find source with google Find source with iqdb
uspenski_apolog_matem_200.gif
>>155209
Начни вот с этой книги.
>> No.155218 Reply
>>155209
Что за причины? Колись давай, не изъёбывайся. Причины у него, лол. На мафию работаешь, что ли? Нахуя конспирация? Тут вниманиеблядей не любят.
>> No.155220 Reply
>>155214
О, спасибо. Мне тут ещё посоветовали "Что такое математика" Куранта. Что скажешь о ней?
>>155218
Да электриком хочу стать. Хотя там физику нужно знать в основном, но математику вспомнить и подучить тоже необходимо. Сам я не совсем гуманитарий - в школе неплохо физику знал, по крайней мере она мне давалась, ну и иностранные языки шарю, хотел одно время на лингвиста пойти, но решил что нахуй нужно и сам изучаю языки.
И, да, если уж такая пьянка пошла - посоветуйте таки по физике что-то подобное.
>> No.155235 Reply
File: xuanwei-su-8593317402-2.jpg
Jpg, 958.91 KB, 1920×1419 - Click the image to expand
edit Find source with google Find source with iqdb
xuanwei-su-8593317402-2.jpg
>>155209
KEVIN HOUSTON,'How to Think Like a Mathematician'.
Самая что ни на есть элементарная СУТЬ, тебе в самый раз как прикладнику. Как читать математику, как ее писать, элементы логики, основные приемы доказательств. В последних главах в качестве тренировки на кошках приведено доказательство нескольких базовых теорем. Английский проще некуда.

Курант не нужен.
>> No.155245 Reply
Поясните, формализация с помощью теории множеств предполагает, что в описании использованы только множества? А где можно почитать, как определены какие-нибудь сложные объекты (без наивности), ведь уже на простых вылезает масса трюков?
>> No.155286 Reply
>>155245
Да, только множества.
В любом учебнике математики.
>> No.155351 Reply
Просьба : помогите с решением задач, пожалуйста.
Задача :
Дан ортоцентрический тетраэдр с ортоцентром h, с центром описаной сферы О с радиусом R и с бимедианой длины a.
а)Доказать что точки О и H симметричны относительно центра тяжести тетраэдра.
б)Доказать что центры тяжести граней лежат на одной сфере.
Буду очень благодарен любой помощи по задачам.
>> No.155352 Reply
>>155351
По сути в задаче просят перевести окружность девяти точек на трёхмерные рельсы. Только как сделать это я не понимаю.
>> No.155446 Reply
File: shift,-rotation-and-simtrans.png
Png, 12.64 KB, 575×54 - Click the image to expand
edit Find source with google Find source with iqdb
shift,-rotation-and-simtrans.png
Понятно, что для любой композиции сдвига и ротации будет одна неподвижная точка, но если от этого еще и гомотетию меньше 1 взять (или вообще любую кроме 1), то каким нахуй образом эта точка останется неподвижной, если только специально её центром гомотетии не взять (но это противоречит условию ЛЮБАЯ композиция).

Вроде на стаке поясняли, но линалом. А тут без линала должна решаться задача, так что нахуй то говнорешение.

Альзо как доказать биноманальную формулу ньютана индукцией без всякой комбинаторики?
>> No.155447 Reply
File: p0337.png
Png, 215.19 KB, 865×1246 - Click the image to expand
edit Find source with google Find source with iqdb
p0337.png
>>155446
> А тут без линала должна решаться задача
С чего бы? Без линала ты даже теорему Шаля не докажешь.

> индукцией
Нужно использовать основное тождество биномиальных коэффициентов.
(n k) = (n-1 k-1) + (n-1 k) - вот оно.

Обосновывается так. Допустим, у тебя есть ящик, в котором n мандаринов. Тебе нужно собрать все k-элементные сочетания мандаринов. Ты берёшь один мандарин и клеишь на него лейбл. Тогда в каждое k-сочетание мандарин с наклейкой либо входит, либо не входит. Те сочетания, в которые мандарин с лейблом входит, получаются выбором k-1 мандарина из n-1 мандарина без этикетки (да ещё мандарин с лейблом, будет k элементов). Те сочетания, в которые мандарин с лейблом не входит, получаются выбором k элементов из n-1 мандарина без этикетки. Так сумма справа и получается.
Если это кажется тебе "нестрогим", докажи как тебе нравится.

Хотя не вижу смысла доказывать бином Ньютона по индукции, потому что пикрелейтед.
>> No.155448 Reply
File: Nbf.png
Png, 4.28 KB, 468×43 - Click the image to expand
edit Find source with google Find source with iqdb
Nbf.png
>>155447
> С чего бы?
Ну до неё никакого линала, а теперь внезапно линал блядь. Это только вступление к матану блджад.
> Нужно использовать основное тождество биномиальных коэффициентов.
> без всякой комбинаторики
Смотри на пик блджад. Написано с многоточием, никаких скобок. Что это тебе говорит?
>> No.155449 Reply
>>155448
> Это только вступление
Это задачи из Зорича. Зорич печально известен своими задачами.
> блджад
Охладите трахание. В стандартном доказательстве бинома Ньютона по индукции используется основное тождество биномиальных коэффициентов. Обойтись без этих коэффициентов ты не можешь, так как они стоят перед слагаемыми в биноме Ньютона. n/1!, n(n-1)/2! - это вот они.
>> No.155451 Reply
>>155449
> Зорич печально известен своими задачами.
Я не против сложности, отнюдь, но блядь как я нахуй должен угадать какие пререквизиты для каждой задачи? Откуда вообще это даунство с паралелльным обучением. Люди должны сначала матан учить, потом линал и только потом физику начинать. Пиздец короче.
>> No.155452 Reply
>>155449
> Это задачи из Зорича. Зорич печально известен своими задачами.
Так эта хуйня вроде и в начале демидовича.
>> No.155453 Reply
Спрошу тогда вот что: нужно ли какую-то дополнительную хуйню знать, чтобы сконструировать биекцию с [0,1] в ]0,1[?
>> No.155454 Reply
>>155451
> как я нахуй должен угадать
Я же не просто так сказал "печально".

>>155453
Нет, не нужно. Даже элементарные функции не нужны. Тут нужно просто додуматься до одной не вполне очевидной вещи. Могу проспойлерить.
>> No.155457 Reply
>>155451
> Люди должны сначала матан учить, потом линал и только потом физику начинать.
Это так не работает.
>> No.155462 Reply
>>155457
Почему же? Работает. Только не в таком порядке, правда.
>> No.155463 Reply
>>155454
А есть что-то, где не печально, кстати?
>> No.155464 Reply
>>155463
Я сам себе задачи придумываю.
>> No.155556 Reply
File: darkness_comes_early_here.png
Png, 1643.94 KB, 1920×1080 - Click the image to expand
edit Find source with google Find source with iqdb
darkness_comes_early_here.png
Доброаноны, необходима ваша помощь, так как я не всё одупляю. Я тупой первокур, и у меня лектор по анализу очень любит всякие логические приколы с действительными числами. Я в последнее время долго думаю над одной задачкой, которую вообще никак не получается решить.

Конкретно суть вот в чём: имеется несколько формулировок непрерывности множества действительных чисел.

1. Существование точки, разделяющей два непустых непересекающихся подмножества R (классика, порой приписывают Дедекинду)
2. Принцип точной верхней (нижней) грани
3. Сходимость ограниченной монотонной последовательности (Вейерштрасс)
4. Все последовательности стягивающихся вложенных отрезков сходятся к некоторой точке R (Кантор)
5. Все фундаментальные последовательности точек R сходятся (Коши)

Берутся любые две формулировки из пяти, требуется доказать их эквивалентность. Некоторые следствия здесь получаются достаточно просто (почти очевидно), но в целом эту задачу я решить не могу до конца. Буду рад, если поможете, ибо это реально кажется мне сложным.
>> No.155557 Reply
File: darkness_comes_early_here.png
Png, 1643.94 KB, 1920×1080 - Click the image to expand
edit Find source with google Find source with iqdb
darkness_comes_early_here.png
Доброаноны, необходима ваша помощь, так как я не всё одупляю. Я тупой первокур, и у меня лектор по анализу очень любит всякие логические приколы с действительными числами. Я в последнее время долго думаю над одной задачкой, которую вообще никак не получается грамотно решить.

Конкретно суть вот в чём: имеется несколько формулировок непрерывности множества действительных чисел.

1. Существование точки, разделяющей два непустых непересекающихся подмножества R (классика, порой приписывают Дедекинду)
2. Принцип точной верхней (нижней) грани
3. Сходимость ограниченной монотонной последовательности (Вейерштрасс)
4. Все последовательности стягивающихся вложенных отрезков сходятся к некоторой точке R (Кантор)
5. Все фундаментальные последовательности точек R сходятся (Коши)

Берутся любые две формулировки из пяти, требуется доказать их эквивалентность. Некоторые следствия здесь получаются достаточно просто (почти очевидно), но в целом эту задачу я решить не могу до конца. Буду рад, если поможете, ибо это реально кажется мне сложным.
>> No.155558 Reply
File: LRhPbh2w5jk.jpg
Jpg, 39.60 KB, 519×604 - Click the image to expand
edit Find source with google Find source with iqdb
LRhPbh2w5jk.jpg
Доброаноны, необходима ваша помощь, так как я не всё одупляю. Я тупой первокур, и у меня лектор по анализу очень любит всякие логические приколы с действительными числами. Я в последнее время долго думаю над одной задачкой, которую вообще никак не получается грамотно решить.

Конкретно суть вот в чём: имеется несколько формулировок непрерывности множества действительных чисел.

1. Существование точки, разделяющей два непустых непересекающихся подмножества R (классика, порой приписывают Дедекинду)
2. Принцип точной верхней (нижней) грани
3. Сходимость ограниченной монотонной последовательности (Вейерштрасс)
4. Все последовательности стягивающихся вложенных отрезков сходятся к некоторой точке R (Кантор)
5. Все фундаментальные последовательности точек R сходятся (Коши)

Берутся любые две формулировки из пяти, требуется доказать их эквивалентность. Некоторые следствия здесь получаются достаточно просто (почти очевидно), но в целом эту задачу я решить не могу до конца. Буду рад, если поможете, ибо это реально кажется мне сложным.
>> No.155559 Reply
Какая-то херня с постингом и айпи произошла. Прошу простить за мультипостинг.
>> No.155565 Reply
>>155556
Уточни вопрос, пожалуйста. Писать двадцать доказательств тебе вряд ли кто-нибудь станет. К тому же пятое нельзя вывести непосредственно из первого, нужны несколько промежуточных лемм.
>> No.155570 Reply
>>155565
К примеру, как доказать равносильность теоремы Кантора и критерия Коши?
>> No.155588 Reply
>>155570
Принцип Коши-Кантора => принцип Больцано-Вейерштрасса.
Пусть последовательность X = x1, x2, ... , xn ограничена. Тогда она целиком содержится в некотором отрезке [a;b]; обозначим его I0. Разделим отрезок [a;b] пополам. Хотя бы в одной из половин лежит бесконечно много точек последовательности. Обозначим эту половину как I1. Определяя по рекурсии, для любого n разделим In пополам и как I(n+1) обозначим какую-нибудь одну из половин, в которой лежит бесконечно много точек последовательности. Таким образом имеем корректно определенную последовательность стягивающихся вложенных отрезков. Пересечение этих отрезков - точка x.
Теперь нам нужно определить подпоследовательность. Сделаем это опять по рекурсии. За y1 возьмём x1. Для каждого отрезка In через y(n+1) обозначим точку последовательности X с наименьшим номером из всех точек, лежащих в отрезке In и отличных от y1, ... , yn. Таким образом, последовательность Y = y1, y2, ... определена.
Докажем, что Y сходится к x. Возьмём какую-нибудь окрестность U точки x. В ней целиком содержится по крайней мере один отрезок IN. Тогда все члены последовательности Y, начиная как минимум с N+1го, лежат в IN и потому в U.

Лемма 1. Фундаментальная последовательность ограничена.
Доказательство. Пусть e = 1. Существует такой номер N, что для всех n,m больших N верно, что расстояние между xn и xm меньше 1. Пусть r - расстояние между xN и x(N+1). По неравенству треугольника для любого n>N расстояние между xn и xN меньше чем r+1.
Пусть r1, r2, ... , r(N-1) - расстояние между xN и соответственно x1, x2, ... , x(N-1). Буквой R обозначим наибольшее из чисел r, r1, ... , r(N-1). Тогда замкнутый шар с центром в xN радиуса R содержит все точки последовательности.

Лемма 2. Если подпоследовательность фундаментальной последовательности сходится, то и вся последовательность сходится к тому же пределу.
Доказательство. Пусть подпоследовательность сходится к a. Рассмотрим шар U радиуса e с центром в a. Рассмотрим другой шар с центром в a радиуса e/2; все точки подпоследовательности, начиная с Nй, лежат в этом шаре, т.е. если n>=N, то расстояние от xn до a меньше чем e/2. Далее, так как последовательность фундаментальна, существует натуральное число M такое, что для любых точек xm,xn с номерами >= M, расстояние между xn,xm меньше чем e/2. Пусть K = max(N,M). Для всех точек с номерами n>=K из неравенства треугольника расстояние от xn до a меньше чем сумма расстояний от xn до xN и от xN до a, т.е. меньше чем e/2 + e/2 = e. То есть все точки последовательности, начиная с Kй, лежат в U. То есть последовательность сходится к a.

Принцип Больцано-Вейерштрасса => критерий Коши.
Пусть последовательность фундаментальна. Тогда она ограничена по лемме 1. Тогда из неё можно выбрать сходящуюся подпоследовательность по лемме Больцано-Вейерштрасса. Тогда вся последовательность сходится по лемме 2.
Обратно, пусть последовательность сходится к a. Тогда в любом шаре радиуса e с центром в a лежат все точки последовательности, начиная с некоторой точки номер N. Тогда расстояние между любыми двумя точками с номерами, большими N, не превосходит диаметра шара, что и означает фундаментальность.

Критерий Коши => принцип Коши-Кантора.
Пусть есть система стягивающихся вложенных отрезков. Через a1, a2, a3, ... обозначим последовательность левых концов отрезков, через b1, b2, b3, ... последовательность правых концов. Рассмотрим последовательность a1, b1, a2, b2, ... Она фундаментальна, так как для любого e>0 существует отрезок номер N такой, что длины всех последующих отрезков меньше e, и потому расстояние между точками последовательности, начиная с точки номер 2N, меньше e. Поэтому она сходится к какому-то числу x. Подпоследовательность сходящейся последовательности сходится к тому же пределу, и поэтому a1, a2, a3, ... сходится к x, b1, b2, b3, ... сходится к x. Предел монотонно неубывающей последовательности не меньше каждого из членов, предел монотонно невозрастающей последовательности не больше каждого из членов, поэтому для любого n выполняется an <= x <= bn, т.е. x принадлежит каждому отрезку. Если пересечению отрезков принадлежит точка y, не равная x, то длины отрезков не могут стать меньше чем |x-y|, так что пересечение в точности равно {x}.
>> No.155725 Reply
>>155558
> Существование точки, разделяющей два непустых непересекающихся подмножества R (классика, порой приписывают Дедекинду)
Но ведь это верно и для Q, в чём подвох?
>> No.155726 Reply
>>155725
Нет, это неверно для Q. Пусть A = {q из Q; q>0 и q^2 < 2}, B = {q из Q; q>0 и q^2 > 2}. Нет рационального числа, которое бы разделяло A и B, потому что разделяющим числом должен был бы являться корень из 2, а он иррационален.
>> No.155734 Reply
Если a=d, то v=c и тождественно s, из чего исходит x>fg
>> No.155738 Reply
File: то.png
Png, 2.80 KB, 275×34 - Click the image to expand
edit Find source with google Find source with iqdb
то.png
Я никогда не умел доказывать теоремы, а тут даже не знаю, как подступиться. Нужно доказать пикрелейтед с помощью алгебры логики. Как это делать?
>> No.155740 Reply
Аноны, у меня тут после окончания школы выдался относительно свободный год,в который бы хотелось подтянуть упущенную в школе математику желательно еще и физику, но по каким книгам этом можно сделать не знаю, так что буду очень благодарен, если кто-нибудь посоветует учебники, желательно еще с задачами/задачниками, которые бы затрагивали курс старшей школы и начала вузовской программы.
>> No.155758 Reply
>>155738
Пусть посылка выполнена.
Пусть x - элемент AxB. Тогда x является парой (a,b), где a элемент A, b элемент B. Но a является элементом C, b - элементом D. Значит, пара (a,b) является элементом и AxD, и CxB.
Обратно, пусть x=(p,q) является элементом и AxD, и CxB. Тогда p является элементом и A, и C; q является элементом и B, и D. То есть p - элемент A, q - элемент B. Тогда пара (p,q) - элемент AxB.
>> No.155760 Reply
>>155740
> но по каким книгам этом можно сделать не знаю
Я на самом деле тоже не знаю. Все книжки или плохие, или очень плохие. Так что бери любой стандартный школьный учебник. Алгебра - Макарычев, потом Колмогоров. Геометрия - Атанасян.
>> No.155770 Reply
>>155740
Пояему бы не зайти на сайт кафедры вуза, в который собираешься поступать, и не посмотреть их программу? Там и список литературы будет.
>> No.155771 Reply
>>155760
> Геометрия - Атанасян
Содержание не соответствует названию. Правильно: Задачки на смекалочку в евклидовом пространстве. Уж лучше почитать книжки Смаллиана или Улама, там хоть интересно.

>>155740
Есть хороший школьный учебник Пратусевича. Также советую Гельфанд,Шень - Алгебра.
>> No.155774 Reply
>>155771
Мне кажется, что ты просто ретранслируешь чужие мемы. Можешь как-то аргументировать своими словами, чем плох Атанасян?
>> No.155785 Reply
Порекомендуйте что-нибудь на тему сюрреальных чисел, не связанное с теорией игр (кроме Кнута).
>> No.155786 Reply
>>155785
Литература есть в википедии. Скачай какую-нибудь книжку и посмотри список литературы в ней.
https://en.wikipedia.org/wiki/Surreal_number
Алсо http://gen.lib.rus.ec/search.php?req=surreal+numbers
>> No.155790 Reply
Аноны, я тут узнал про оригами-построения, и был обрадован и удивлен. Стали очень любопытны две вещи:
1) Какие еще существуют интересные инструменты для построений?
2) Можно ли рассматривать оригами-построения не на плоскости, а на какой-то другой поверхности? Понятно, что для возможности образования складки необходима нулевая кривизна, но возможно существуют какие-то обобщения?
>> No.155791 Reply
>>155790
Вопросы хорошие, но тема уж очень специфическая.
>> No.155806 Reply
File: Screenshot-from-2016-11-15-20-18-12.png
Png, 181.29 KB, 709×196 - Click the image to expand
edit Find source with google Find source with iqdb
Screenshot-from-2016-11-15-20-18-12.png
>>155774
Конечно. Во-первых он содержит множество наивностей, а аксиоматика приводится только в конце учебника в совершенно маразматических формулировках. Совершенно очевидно, что даже это дополнение в учебнике только для галочки, потому что у аксиом нет никакой мотивировки и даже слово аксиома принимается в каком-то разговорном смысле. Это, наверное, самое ужасное, в учебники по геометрии отсутствует аксиоматический метод. Нет, мы не задаем себе аксиомы для удобной работы, мы просто верим в какие-то правила, которые внезапно зависят от пространства. Ну и про раздел с векторами лучше ничего не говорить, благо она хоть есть.
Во-вторых учебник рассчитан на невероятное количество часов геометрии. В каждой главе в среднем по 30 задач. Уж столько ли необходимо для прохождение таких содержательных тем как "перпендикулярность прямых в пространстве"? При этом в большинстве свои задачи простой ригоризм на тему вывода тривиальных рассуждений. Что забавно, без аксиоматического метода совершенно неясно, что интуитивно, а что нет. Вот надо доказать теоремы о перпендикулярах между параллельными прямыми, о сумме углов треугольника, пользуясь аксиомой о параллельных прямых. Но все три утверждения на самом деле равнозначны. Но даже не в этом дело. Большинство задач содержат один или два хода. В большинстве задач сложность именно в том, чтобы записать доказанное. При этом самое удивительное, что ученики учили геометрию и до 10 класса. И неужели они не знают о свойствах параллельных прямых?
Далее претензия касательно содержания учебника: причем тут вообще геометрия? Материал за исключением векторных глав вполне покрывается "началами евклида". Такова была математика Эллады. Но время шло, наука менялась, зачем же давать настолько подробное изложение архаического метода? При том, что с координатным методом школьников как бы обкрадывают. Они долго и упорно комбинаторно решают всякие задачи на вычисление какой-то стороны или площади, будто запрещено использовать вектора, надо искать больше подобных треугольников! а потом им как кость кидают метод, где все решает в два прихлопа два притопа, но не объясняют всей его мощности, позволяющей сводить все к несложной алгебре. Где понятие векторного пространства в главе про векторы? Даются теоремы Чевы и Менелая. Где хоть слово про афинное? В учебник полным полно странных задач про проекции, зачем же мы проецируем, чтобы не узнать про проективное пространство? Полезно, конечно, для некой интуиции, знать некоторые интересные свойства плоских фигур, но почему этому посвящен весь учебник? Но тогда он и должен называться "методы решения стереометрических задач".
По мне причина такого ужаса весьма очевидна, ведь учебник писали на скорость, бездумно, держа в голове модель идиотов учитель и их дебилов учеников, чтобы можно было хоть методом разучивания теорем наизусть "учить" геометрии. Потом также бездумно учебник дополнили более сложными главами, которые находятся где-то сбоку, ведь материал там выглядит совершенно потерянным и оторванным от остального, заодно напихали в конец каждой главы задач из сборников городским олимпиад, чтобы "место знали". Спросит учителя кто-то: а почему так легко? Она ему раз и гроб какой-нибудь с областной олимпиады! Далее в учебник приписали после нытья каких-нибудь математиков несколько околострогих фраз, впихнули аксиомы в каком-то издании "чтобы были" и готово. Учиться по такому крайне не рекомендуется.
>> No.155807 Reply
>>155790
1 спирограф, например
2 можешь погуглить на тему non flat origami
>> No.155808 Reply
>>155806
Я не тот, кто у тебя об этом спрашивал, но тоже говорю спасибо за, гхм, пояснение по хардкору.
>> No.155818 Reply
File: 1302412313870.jpg
Jpg, 55.10 KB, 500×500 - Click the image to expand
edit Find source with google Find source with iqdb
1302412313870.jpg
В чем разница между кронекеровым, внешним и тензорным произведениями?
>> No.155820 Reply
>>155818
> кронекеровым
Операция над матрицами
> внешним и тензорным произведениями
Операции над векторами.
> внешним
v1⊗v2 - v2⊗v1
>> No.155826 Reply
>>155820
Есть разные внешние произведения. Я говорю про то, которое дает матрицу, условно вектор-столбец на вектор-строку. Является ли оно тензорным произведением этих векторов? Является ли кронекерово произведение тензорным произведением двух матриц?
>> No.155980 Reply
File: IMG_2031_cr.jpg
Jpg, 898.42 KB, 2587×728 - Click the image to expand
edit Find source with google Find source with iqdb
IMG_2031_cr.jpg
Суп, добрач!
Начал тут недавно освежать в голове материал по математике, который проходил в техникуме. Наткнулся в своей тетради на пикрилейтед.
Никак не вкурю как здесь можно выносить ab за скобки в числителе. Это вообще легально? Если да, то как? Чет у меня через обратное действие не получается исходного вида.
>> No.155983 Reply
>>155980
По-моему у тебя тут ошибка.
>> No.155984 Reply
>>155980
> а^(1/3)b - ab^(1/3) = а (a^(-2/3)b - b^(1/3)) = ab (a^(-2/3) - b^(-2/3))
http://www.wolframalpha.com/input/?i=expand+ab+*+(a%5E(-2%2F3)+-+b%5E(-2%2F3))
>> No.155990 Reply
>>155983
>>155984
Спасибо!
Теперь понятно.
>> No.156028 Reply
>>155826
Если оно линейно по каждому аргументу, то почему бы и нет?
>> No.156048 Reply
>>155818
тензорное произведение - пожалуй самая общая операция умножения векторных пространств. Удобно понимать дуальное к нему пространство как пространство всех линейных по обоим аргументам функций.
кронекерово - реализация тензорного в случае матриц, когда ты хочешь явно работать с конкретными базисами.
внешнее- антисимметризация тензорного, или дуальное к нему пространство - функции линейные по обоим аргументам и антисимметричные (меняющие знак при перестановке)
>> No.156345 Reply
Как решать уравнения вида:
a(f1(t))+b(f2(t))+c(f3(t)=f4(t), по t, где fn(t) некий приводимый в R многочлен, типа x^2+2x+1.
>> No.156359 Reply
>>156345
Не знаю.
>> No.156361 Reply
>>156359
Ну я интуитивно догадываюсь, что при каждом t одно из согласных становится нулем, а уравнение вид ax+by=z, которое очевидно имеет решение при взаимно-простых a и b. Только я не могу осмыслить, можно ли получить таким способом все решения.
>> No.156377 Reply
>>156361
Тебе нужно решить какое-то конкретное уравнение или нужен метод для любых констант? Решать относительно t?
>> No.156381 Reply
>>156377
Решать относительно t. Нужен метод: что-то получше, чем просто перебор.
>> No.156387 Reply
>>156381
В смысле перебор? Это как?
>> No.156388 Reply
>>156387
Стоит дополнить, что метод очень простой и называется в лоб. Я даже дам подсказку, но только после того, как ты ответишь на вопрос.
>> No.156404 Reply
>>156387
Подставляем корень f1 как t, смотрим решается ли уравнение, подставляем другой корень, снова смотрим, etc...
>> No.156423 Reply
>>156404
Я дико извиняюсь, но что нам мешает подставить корень такого уравнения:
x^2+5x-6=0
Если вы не поняли моей шутки, то спрошу иначе: как корни многочлена связаны с решением задачи?
>> No.156424 Reply
>>156423
пусть уравнение такое, например:
a(x^2+5x-6)+b(x^2-4)+c(x+3)=x
Тогда при x=1 уравнение примет вид
-3b+4c=1, следовательно x=1 является решением. Если бы было бы что-то типа -6b+4c=1, то x=1 не является решением.
>> No.156473 Reply
File: mathmathmath.png
Png, 37.88 KB, 872×840 - Click the image to expand
edit Find source with google Find source with iqdb
mathmathmath.png
Доброанон, выручи, помоги хоть что-то первокуру решить.
>> No.156478 Reply
>>156424
хотя я какую-то пургу несу, решать то надо не в Z, а в R(t).
>> No.156479 Reply
>>156424
Насколько я понял, a b c должны быть заданы с начала. Но видимо я ошибаюсь. Как ваше решение связано с тм, над чем нужно решить не понятно. В общем сформулируйте лучше задачу, что дано, что найти.
>> No.156485 Reply
Реши методом дихотомии и все епта. Либо методом Ньютона, еще пиздачей.
>> No.156496 Reply
>>156479
Что дано, это a,b,c и приводимые многочлены от x в качестве коэффициентов. Надо найти решение в кольце многочленов R[x].
>> No.156501 Reply
>>156473
Это у тебя в первом семестре преподают весь курс анализа? Кек
>> No.156517 Reply
File: vQxonIQrNEw.jpg
Jpg, 35.98 KB, 400×400 - Click the image to expand
edit Find source with google Find source with iqdb
vQxonIQrNEw.jpg
>>156501
Щас бы давать интегралы второго рода на первом семестре.
>> No.156519 Reply
>>156496
Теперь я совсем запутался. Нужно было же найти x удовлетворяющий равенству. Теперь решение лежит в алгебре многочленов. И что это значит?
>> No.156520 Reply
Как вспомнить всё и пройти не понятое за 7-11 классы школы в кратчайшие сроки (допустим, 3 месяца)? Какой учебник порекомендуете?
>> No.156547 Reply
>>156501
Да, я вообще экономист и нам всё запихнули в один семестр. При этом почти ничему так и не научили. В теории вероятности сказали разобраться самим.
>> No.156566 Reply
>>156520
interneturok.ru
>> No.156571 Reply
Объясните не-математику чем примечательны аналитические функции. Нет, у меня не сессия или собеседование -- просто любопытно. Вся соль в том, что их можно разложить в ряд Тейлора? Если так, то что это дает? Наглядность как связаны переменные?
>> No.156574 Reply
>>156571
Класс аналитических функций замкнут относительно сложения, умножения и композиции. Любая аналитическая функция бесконечно дифференцируема. У аналитических функций много хороших свойств. С помощью теории аналитических функций мы можем легко и просто, с помощью ряда, определять новые функции, которые будут аналитическими и, значит, обладать кучей хороших качеств. Например, мы легко можем определить экспоненту e^z, синус, косинус - все они будут дифференцируемы из коробки, нам не придётся доказывать их гладкость отдельным рассуждением.
>> No.156580 Reply
>>156547
Тебя, как экономиста, учили 1 семестр интегралам, а в тервере сказали "разобраться самим"? Вот уж воистину туземное образование, слов нет.
>> No.156584 Reply
>>156574
Но что это за хорошие свойства и качества?
>> No.156586 Reply
>>156584
Тебе мало что что ты можешь дифференцировать этик сучек сколько пожелаешь? Тогда посмотри, как они ведут себя на комплексной плоскости!
У функций определенных рядом только на одном участке есть единственно аналитическое продолжение на всю плоскость. Вот в этом видео неплохо изложен смысл аналитического продолжения для не-математиков:
https://www.youtube.com/watch?v=sD0NjbwqlYw
>> No.156588 Reply
>>156586
> Тебе мало что что ты можешь дифференцировать этик сучек сколько пожелаешь?
То есть это по-сути, единственное, что меня интересует?

> https://www.youtube.com/watch?v=sD0NjbwqlYw
Вот я задался этим вопросом именно после этого ролика. В той же дзета-функции, область определения - реальная часть > 1, но если использовать аналитическое продолжение, то получаются такие "сказочные" штуки как 1 + 2 + 3 + ... = -1 / 12.
>> No.156589 Reply
>>156588
Смысл аналитического продолжения заключается не в "сказочных штуках", а в том, что если у тебя есть аналитическая хоть где-то функция, то эта же функция единственным образом продолжается сразу на все комплексные числа. Это просто невероятно мощное и очень неочевидное свойство. Как если тебе дают винтик, и ты автоматически получаешь из него ракету. И ты не можешь получить из него ничего другого, он содержит в себе всю информацию об этой ракете, и только о ней!
>> No.156590 Reply
>>156588
> То есть это по-сути, единственное, что меня интересует?
А лёгкое нахождение некоторых интегралов тебя интересует?
> получаются такие "сказочные" штуки как 1 + 2 + 3 + ... = -1 / 1
А потому, что не надо тупо подставлять значения туда, куда не следует.
>> No.156592 Reply
>>156589
Спасибо за такое сравнение!
Получается, если
> он содержит в себе всю информацию об этой ракете, и только о ней
Но эта информация содержит такие штуки как -1/12, это значит что мы чего-то пока не понимаем?
>>156590
> А лёгкое нахождение некоторых интегралов тебя интересует?
Наверное, да? Если я физик, например?

> А потому, что не надо тупо подставлять значения туда, куда не следует.
Если глянуть на тематические видео на ютубе (не лучшее место, я знаю), то кажется что тру-математикам сложно удержаться и не показать этот пример.
>> No.156593 Reply
>>156592
> Если глянуть на тематические видео на ютубе (не лучшее место, я знаю), то кажется что тру-математикам сложно удержаться и не показать этот пример.
Научпоп тяготеет к охуительным историям. Математики же обычно этого не делают.
>> No.156594 Reply
>>156592
> Но эта информация содержит такие штуки как -1/12, это значит что мы чего-то пока не понимаем?
Сомнительно. Анон выше правильно сказал, что это получено подстановкой чисел куда не следует. Если выражение не имеет смысла, то из него как правило можно получить все что хочется.
>> No.156595 Reply
Сколько корней имеет x^26+1 в поле из 27 элементов?
Я дошел до того, что либо ни одного, либо 26, если поле состоит из нуля и обратных самим себе по сложению элементов. Возможно ли такое?
>> No.156596 Reply
>>156595
Подумай. Сколько элементов в мультипликативной группе этого поля? Как соотносятся порядки группы и ее элементов? Как соотносятся -1 и 1 в этом поле?
>> No.156598 Reply
>>156594
> Анон выше правильно сказал, что это получено подстановкой чисел куда не следует.
Но это получено через аналитическое продолжение? Почему туда не следовало подставлять числа?

> Если выражение не имеет смысла, то из него как правило можно получить все что хочется.
как здесь? https://www.youtube.com/watch?v=-EtHF5ND3_s
>> No.156599 Reply
>>156596
26 эл-ов в группе. Эта группа циклическая. Это все участвовало в моем решении. Тогда получается, что x^26 это всегда 1. Однако 1+1 может быть равно 0. Или не может, я не вижу противоречий, в этом то и вопрос.
>> No.156600 Reply
File: рамануджан.png
Png, 48.99 KB, 1033×243 - Click the image to expand
edit Find source with google Find source with iqdb
рамануджан.png
>>156598
Это суммирование по Рамануджану. Используется в созданной Рамануджаном науке и нигде больше. Граничит с научным фричеством, пикрелейтед.
>> No.156601 Reply
>>156599
> Однако 1+1 может быть равно 0
Поразительно, ты фейлишься на самой простой части. Характеристика у поля какая у тебя? Почему в поле 27 элементов, а не 26 или 29?
>> No.156602 Reply
>>156601
Я тебя не понимаю. 27 элементов, потому что в задаче так написано. В непонятном для меня случае характеристика равна 2, получается.
>> No.156603 Reply
>>156602
А если бы в задаче было написано, что элементов 6? Выпиши-ка сюда список всех элементов поля GF(6).
>> No.156604 Reply
>>156603
Ты имеешь в виду, что моя мультипликативная группа изоморфна сумме циклических групп? Я так понимаю, это означает, что если каждый эл-нт в мультипликативной группе поля обратен себе, то порядок поля это степень двойки? Я правильно все понял? Тогда это дает противоречие, а значит таких полей не существует.
>> No.156605 Reply
>>156604
> Ты имеешь в виду, что моя мультипликативная группа изоморфна сумме циклических групп?
ОМФГ, нет конечно, мультипликативная группа конечного поля всегда циклическая.
Я имею в виду, что ты пытаешься решить задачу об объекте, о свойствах которого ты не имеешь ни малейшего представления. Мне не хочется тебе давать рыбу, но размахивание топором в попытках навести твои мысли на изготовление удочки кажется начало приводить к обратному эффекту. Поставлю вопрос в лоб: какое количество элементов в принципе может быть в конечном поле? На этот счет есть теорема, без которой любая работа с конечными полями вообще бессмысленна, ибо она определяет их структуру.
>> No.156607 Reply
>>156605
Ага, степень простого числа. Упустил этот момент. Но как это связано с моей задачей?
>> No.156608 Reply
>>156605
p^n, где p - простое число. И что? 27=3^3.
мимошёл
>> No.156609 Reply
>>156607
Ну даже не знаю. Может быть есть какая-то связь между простым подполем и характеристикой поля, например?
>> No.156610 Reply
>>156607
Видимо, он хочет сказать, что в твоём случае характеристика равна 3. Характеристика - это то простое число, степень которого является порядком.
>> No.156611 Reply
>>156609
Понятия не имею какая связь, быстрое гугление не помогло.
>>156610
Почему эту простую фразу так мутно написали в википедии. Ну тогда все получается.
>> No.156612 Reply
>>156610
Формально нет, характеристика - это порядок единицы в аддитивной группе. То, что он равен порядку простого подполя, является следствием того, что любое конечное поле изоморфно полю многочленов по модулю неприводимого.
>> No.156613 Reply
>>156609
>>156612
Самое интересное, что в лекциях по которым я иду ничего этого нет. Есть теорема без доказательства, что поле p^n есть и единственно с точностью, до изоморфизма. А как задачу, то мне решить?
>> No.156614 Reply
>>156598
> Но это получено через аналитическое продолжение? Почему туда не следовало подставлять числа?
Аналитическое продолжение функции, заданой с помощью ряда. Не продолжение самого ряда или понятия бесконечной суммы.

Вообще, есть распространённая проблема: люди путают синтаксис и семантику, из-за чего появляются 1 + 2 + 3 + ..., деление на нуль, 0.(9) и прочие удивительные истории. Лечение стандартное: спрашивать, спрашивать и ещё раз спрашивать. А именно: какой же великий смысл имеют сей тайный набор символов? Какой смысл несёт этот набор священных слов? Что означают эти странные картинки?
Потому, что заклинание «нестандартный анализ!» не спасёт от необходимости пояснить, что же такое 0.(9). И пока соответствующее определение не дано, конструкции вроде 0.(9) ≠ 1 (как и любые другие конструкции, включающие 0.(9)) — бессмысленная хуита.
>> No.156615 Reply
File: Le_songe_de_la_raison.jpg
Jpg, 130.44 KB, 450×668 - Click the image to expand
edit Find source with google Find source with iqdb
Le_songe_de_la_raison.jpg
>>156614
Пик отклейился.

к: которые подвал
>> No.156616 Reply
>>156613
> в лекциях по которым я иду ничего этого нет
Значит по этим лекциям нельзя изучать конечные поля. Вообще. Совсем. Стандартный совет: Лидл-Нидеррайтер.
> А как задачу, то мне решить?
Ты уже сделал все нужные шаги. Осталось только ответить на вопрос равняется ли 1 и -1 в поле характеристика 3. Нет, не равняется.
>> No.156617 Reply
File: 14632415690901.jpg
Jpg, 175.33 KB, 800×1200 - Click the image to expand
edit Find source with google Find source with iqdb
14632415690901.jpg
>>156616
Ну это я понял, не совсем тупой. Теперь пойду изучать почему поле характеристики три со всеми доказательствами. Спасибо.
>> No.156626 Reply
>>156614
Вообще изначальный вопрос был "Что такого в аналитических функциях". Ответами проследовало: удобно дифференциировать, удобно интегрировать, аналитическое продолжение. Пока что не совсем понял про продолжение, например, значит ли это что к любым значениям продолжения не стоит относится серьезно, или только к каким-то конкретным значениям (а если так, то что это за ущемление некоторых чисел?) Если это в общем-то все, то ретируюсь к остальному науч-попу на ютубе. Хотя мне теперь интересно >какой же великий смысл имеют сей тайный набор символов?
но мне кажется, что про семантику уже где-то видел на той же трубе.
>> No.156631 Reply
>>156626
Значения аналитического продолжения не являются значениями исходной функции, хотя и полностью задаются ей. Если функция задана через ряд, то подстановка в значений в ряд имеет смысл только там, где ряд сходится. Однако, этот же ряд задает значения аналитического продолжения везде, даже там где сам ряд перестает "работать". Это хорошие, годные значения, которые ничем не хуже значений, полученных подстановкой чисел в ряд, но их ты такой подстановкой получить не можешь. Аналитическое продолжение функции f - это другая функция, которая совпадает с f на ее области определения. Однако так как оно единственное, нет смысла рассматривать их раздельно.
Ну и вообще, если хочешь действительно понять, а не вот так через всякие аналогии, то лучше возьми какой-нибудь учебник по комплексному анализу. Аналогии - вещь довольно опасная, когда не понимаешь что лежит внизу. Некоторые вон вообще научными фриками становятся, потому что им не нравится объясненная на пальцах теория относительности, а настоящую они даже и не пытались никогда изучить.
>> No.156665 Reply
Какие цели преследуете занимаясь математикой?
>> No.156666 Reply
>>156665
Это как наркотик. Раз начав, слезть нельзя.
>> No.156667 Reply
>>156665
1. Язык (физики)
2. Красиво же!
>> No.156682 Reply
>>156631
Ну вот это объяснение было довольно доступным и понятным. В учебниках, обычно, все описано очень абстрактно, что-то понять (именно понять, а не узнать формулировку, и например заучить) - редко получается.
>> No.156698 Reply
File: 14696262391550.jpg
Jpg, 37.98 KB, 368×539 - Click the image to expand
edit Find source with google Find source with iqdb
14696262391550.jpg
Новый день, новые непонятки.
Изоморфны ли фактор кольца
Z3[x]/x^2+x и Z3[x]/x^2+x+1
Я до этого встречался с задачами только про изоморфность над кольцом R[x] по многочлену и то знаю, как делать только если они не изоморфны, а в одном есть делитель нуля, то есть в случае с R[x]/q(x), когда корни многочлена лежат в комплексной плоскости, а значит не существуют делители нуля в факторкольце, а в другом все корни лежат в R, значит делители существуют. Помогите со случаем Z[3]. Почитал вики про факторкольца, но не понял как применить в моем случае.
>> No.156711 Reply
>>156698
> Почитал вики
Нельзя изучать математику по википедии.
x^2+x+1 = (x-1)^2 <- идеал, порожденный этим многочленом лежит в идеале, порожденном (x-1)
x^2+x = x(x+1) <- произведение двух разных идеалов
Выходит совершенно разная структура идеалов, и как следствие, фактор-колец. Однако, не стоит мне верить на слово, я бы порекомендовал проверить это руками, выписать все элементы фактор-колец и посчитать их порядки, благо многочлены совсем маленькие.
>> No.156741 Reply
>>156711
Ну как немного. Я так понимаю будет 12 элементов, а потом нужно будет еще считать порядки для каждого. Неужели нет более весомого противоречия?
>> No.156742 Reply
>>156741
> весомого
простого
Фикс.
>> No.156743 Reply
>>156741
Откуда ты взял 12? Я так понимаю, у тебя какие-то проблемы с выписыванием элементов фактор-кольца. Это всего лишь значит что ты не понимаешь, как оно устроено. В явном виде их выписать - это лучший способ это понять. Иначе у тебя в сознании не будет абсолютно никакой связи между многочленами, идеалами и фактор-кольцами. Ты понять хочешь или решить задачу на отъебись для препода? Если второе, то ты не туда зашел.
>> No.156744 Reply
>>156743
Пардон, пересчитал, 27 многочленов 0, 1, 2 степени. Вот тут то я и не понял, в каждом элементе фактор множества должно содержаться два различных многочлена, но тогда их должно быть четное число. Какой-то многочлен должен не иметь другого эквивалентного, например по модулю (x+1)x, но с другой стороны каждый такой имеет, причем ищется он по вполне очевидному алгоритму.
>> No.156745 Reply
>>156744
Новый фикс, не два а три, я понял свою ошибку, выписал, спасибо, вроде понятно.
>> No.156746 Reply
>>156744
>>156745
Не уверен, что тебе понятно. В нем нет многочленов второй степени. Элементы фактор-кольца - это по сути остатки от деления на твои многочлены. Разница только в том, что в фактор-кольце как оно определено каждому из остатков вообще говоря соответствует класс эквивалентности вида остаток+k*многочлен
>> No.156747 Reply
Выпиши их сюда. А то черт знает, куда там тебя твоя википедия заведет, а нормальные книжки ты читать не хочешь. А еще я через час съебываю на самолет и более не смогу тебе помогать.
>> No.156748 Reply
>>156746
Да это я знал.
>>156747
Еще не досчитал, но их должно быть 9 штук.
>> No.156749 Reply
>>156747
Раз сваливаешь, то давай на перед обсудим мои действия. Вот я выписал все элементы для этих двух многочленов. Как я их найду дело мое(переберу все многочлны 0 1 и 2 степени и найду эквивалентные), после этого что я должен буду сделать?
>> No.156751 Reply
>>156749
Зачем тебе перебирать многочлены второй степени? Остаток от деления на многочлен второй степени не может иметь вторую степень. Только строго меньше.
На счет эквивалентных... В принципе ищи, это улучшит твое понимание предмета.
После того как ты их выпишешь просто поставь галочки на делителях нуля. Делители нуля прямо связано с корнями (вернее, разложением) многочлена, который порождает идеал.
>> No.156752 Reply
>>156747
Ура я понял, в обоих случая это будут все многочлены первой и нулевой степени.
>> No.156753 Reply
>>156752
Именно так. А разница между кольцами - в таблице умножения в них. Для того, чтобы показать что они неизоморфны достаточно найти одно ее свойство, которое различается. В твоем случае это разное количество делителей нуля. Также это может быть разное количество элементов заданного порядка, или отсутствие элементов определенного порядка в одном из колец.
>> No.156754 Reply
>>156753
Да, прошарилтеперь все понятно. Построить факторгруппы было отличным советом.
>> No.156766 Reply
А вот еще мне попалось такое задание:
сколько корней имеет x^28 над полем из 27 элементов. Я сказал, что если g - порождающий, то x=g^y; g^2y=-1, тогда, g^2(13-y)=-1 и g^2(13+y)=-1, т.е. 3 корня, противоречие. Единственная, что мне не нравится y=0, тогда второй и третий корни равны и доказательство не работает. Можно сказать, что в таком случае 1=-1, но тогда нужно обосновать почему поле из 27 элементов имеет характеристику 3.
>> No.156767 Reply
File: IMG_20161225_154918.jpg
Jpg, 85.74 KB, 576×549 - Click the image to expand
edit Find source with google Find source with iqdb
IMG_20161225_154918.jpg
Ребятки, помогите с заданиями.
Во втором только с поверхностью непонятно.
>> No.156785 Reply
>>156519
ну представить его линейном виде y = x (t) + z (t)+c, например.
>> No.156800 Reply
>>156785
Как x связано с x в алгебре многочленов? Почему x() и z() нельзя сложить? Что еще за y? Короче давай еще раз, забыли все написанное выше, сформулируй задание.
>> No.156833 Reply
>>156800
есть диофантово уравнение x(t)+y(t)+z(t)=c в кольце многочленов, пусть Z(t). Надо описать все его решения.
>> No.156985 Reply
В автокатастрофе погиб один из создателей Московского центра непрерывного математического образования, Летней школы в Дубне, известный организатор олимпиад школьников и внешкольного математического обучения Виталий Арнольд.
>> No.156996 Reply
>>156985
Сначала подумал, что тот Арнольд умер, а потом вспомнил, что он умер ещё 6 лет назад.
>> No.157026 Reply
Аноны, есть одна задача.
Нужно описать спираль в полярных координатах. Спираль выглядит как если прямую "намотать" на вертикальный цилиндр (вертикальная координата всегда возрастает), но с одним отличием. Отличие в том, что вертикальная координата точек спирали периодически убывает, но так, что спираль никогда не пересекается сама с собой. Например, пока угловая координата лежит в пределах [0; PI], вертикальная координата возрастает, от PI до 2*PI - убывает, но не настолько сильно, чтобы спираль пересекла саму себя.
Мне нужна функция, описывающая эту кривую в цилиндрических координатах.
Параметры:
t - угловая координата, p - период возрастания вертикальной координаты, l - коэфициент изменения вертикальной координаты (за сколько t насколько изменится вертикальная координата), k - разинца между скоростью возрастания и убывания вертикальной координаты (чтобы при убывании спираль не пересекла саму себя).
>> No.157033 Reply
>>157026
Картинку-то нарисуешь?
>> No.157047 Reply
С этим >>157026 разобрался, но возникло еще два вопроса.

1. Как называется трехмерная фигура в виде восьмёрки, не пересекающей саму себя? Что-то примерно такое:
{
x = r * cos(t);
y = sin(t);
z = r sin(t)cos(t);
}
2. Есть произвольная прямая в пространстве, не параллельная ни одной из осей координат. Есть координаты точки на плоскости, перпендикулярной к этой прямой, причём координаты даны относительно точки пересечения плоскости с кривой. Нужно описать процедуру пересчета координат этой точки относительно начала координат.
>> No.157058 Reply
File: 14608444328200.jpg
Jpg, 31.37 KB, 337×513 - Click the image to expand
edit Find source with google Find source with iqdb
14608444328200.jpg
>>157047
Я чего-то не понимаю.
Рассмотрим трехмерное пр-во. Пусть есть некие координаты относительно плоскости перпендикулярной прямой(координаты по одной точке не строятся, нужны оси, а если их выбирать произвольно, то очевидно, что они могут сильно меняться, с точностью до окружности). Теперь сдвинем нашу плоскость вдоль прямой вниз или вверх, точка пересечения пускай будет та же, тогда данные нам координаты точки не поменяются, в отличие от координат этой точки в пространстве.
>> No.157063 Reply
File: 14575336920470.jpg
Jpg, 9.52 KB, 304×323 - Click the image to expand
edit Find source with google Find source with iqdb
14575336920470.jpg
>>156996
А я все эти 6 лет думал, что 6 лет назад умер этот Арнольд.
>> No.157064 Reply
>>157047
> причём координаты даны относительно точки пересечения плоскости с прямой
Слоуфикс
>>157058
А вот как быть с выбором осей, я даже не представляю. Какие возможны варианты?
>> No.157146 Reply
Я нашёл удивительную книгу, где основные понятия дифференциальной геометрии изложена просто и понятно, да ещё и с физическими приложениями. Жаль, что она не попалась мне раньше: Б. Шутц «Геометрические методы математической физики».
>> No.157147 Reply
>>157146
Добавил в закладки, спасибо.
>> No.157181 Reply
Привет всем, сейчас тополог попросил предъявить содержательный пример непрерывной биекции мкжду двумя топологическими пр-вами так, чтобы обратная к ней непрерывной не была. Желательно чтобы оба пр-ва были подмножествами прямой со стандартной топологией. Я ничего такого придумать не смог. Может кто знает примеры?
>> No.157182 Reply
>>157181
Тождественное отображение из R с дискретной топологией в R со стандартной топологией.
id: Rд -> Rст очевидно непрерывно, прообраз открытого множества открыт.
id: Rст -> Rдискр не непрерывно. Прообраз любого одноточечного множества не открыт.
>> No.157193 Reply
>>157182
Спасибо, но мне объяснили, что смену топологий я могу себе в жопу засунуть и если уж тождественное отображение не является функцией то я могу себе его засунуть по соседству с топологиями. От меня хотели что-нибудь содержательного, а до дискретной и антидискретной топологии по одному пространству я и сам додумался. Именно поэтому я написал, что оба топологических пр-ва должны иметь одинаковую топологию, т. е. быть подмножествами одного пр-ва. Также было получено указание найти такое отображение на R со стандартной топологией.
Подытожим, чтобы вопросов больше не возникало:
f:X->Y, X,Y – подмножества R со стандартной топологией.
>> No.157198 Reply
>>157193
Пусть A = [0;1), B = [2;3], X = объединение A и B.
Положим f(x) = x, если x элемент A, и f(x) = x-1, если x элемент B.
Она биективно отображает X на [0;2]. Она непрерывна, но не гомеоморфизм.
>> No.157199 Reply
File: 14841386580360.png
Png, 661.89 KB, 620×800 - Click the image to expand
edit Find source with google Find source with iqdb
14841386580360.png
>> No.157202 Reply
File: вуц.png
Png, 2.60 KB, 316×70 - Click the image to expand
edit Find source with google Find source with iqdb
вуц.png
>> No.157212 Reply
>>157198
Спасибо, действительно хороший пример, я думал о чем-то похожем, но не догадался.
>> No.157229 Reply
>>157202
Пусть f и g - функции из объединения B и C в A. Значками f1 и f2 будем обозначать сужения f соответственно на B и на C. Аналогично с g1 и g2.

Ясно, что если f не равно g, то не может быть так, что f1=g1 и f2=g2; обязательно или f1 != g1, или f2 != g2, или оба случая сразу. Поэтому отображение, сопоставляющее каждой функции f пару (f1, f2), обязательно инъективно. В одну сторону инъекция есть, таким образом.

Обратно, рассмотрим пару (f1, f2), где f1 - функция из B в A, и f2 - функция из C в A. Определим функцию f, положив f(x)=f1(x), если x элемент B, и f(x)=f2(x), если x элемент C. Для любой другой пары (g1,g2) аналогично определенная функция g будет не равна f. Поэтому отображение, сопоставляющее каждой паре (f1,f2) указанным образом определенную функцию f, инъективно. Таким образом, инъекция есть и в другую сторону.

Множества равномощны по теореме Кантора-Бернштейна.
>> No.157279 Reply
Кто-нибудь может пояснить за теорию Лефшеца? Больше всего интересует формула Пикара-Лефшеца и гиперплоские сечения. Бэкграунд больше геометрический, знаком с теорией Морса.
>> No.157280 Reply
>>157279
Я не могу.
ОП
>> No.157285 Reply
Доброанончики, а что вы думаете по поводу математического раздела сами-знаете-где?
>> No.157286 Reply
>>157285
Как человек, стоявший у его истоков, весьма стыжусь. Но что сделано, то сделано.
>> No.157289 Reply
>>157286
Стыдишься чего? Школьников и х-а? Или просто общей атмосферы? Там вроде поживее, чем тут.
>> No.157290 Reply
>>157289
Того рака, который получился в результате. И его, да.
>> No.157304 Reply
>>157289
Здесь лучше хотя бы потому, что анон лучше отвечает.
>> No.157305 Reply
File: 2017-01-18-164640_1366x768_scrot.png
Png, 149.51 KB, 1298×693 - Click the image to expand
edit Find source with google Find source with iqdb
2017-01-18-164640_1366x768_scrot.png
Помогите понять определение и следующее за ним упражнение. Что такое граф я понял, а вот как ввести на нем топологию вообще не ясно. Если я правильно понял написанное, берется граф и то ли стягивается (как гомотопией) в набор букетов окружностей (каждой соответствует компонента связности начального графа), то ли склеивается как-то, вкладывается в R^2 и там топология индуцируется с R^2. В общем, если кто-то нарисует что происходит или внятно расскажет -- буду очень благодарен.
>> No.157307 Reply
>>157305
Графу сопоставляется подмножество в R^3 и рассматривается индуцированная топология.
1. Для каждой вершины графа берем точку в R^3.
2. Если две вершины графа соединены ребром, соответствующие точки соединяем линией.
Топологический граф - это не более чем рисунок абстрактного теоретико-множественного графа. Топология индуцируется.
>> No.157308 Reply
>>157307
А почему именно R^3? Я не очень силен в теории графов. Знаю, что не все графы вкладываются в R^2, а в R^3 любые, значит?

Всё остальное понял, спасибо!
>> No.157309 Reply
>>157304
Ну здесь атмосфера другая и масштаб другой, разумеется. Да и доброчан все-таки. Если что, про хуже\лучше речи и не шло, я этого не имел в виду.
>> No.157312 Reply
>>157308
> а в R^3 любые, значит?
Да, и это даже нетрудно доказать.
>> No.157323 Reply
>>157312
Точно, почитал про книжные вложения.
>> No.157325 Reply
Я смотрю на дифференциальную геометрию. Многообразия, координатные базис, дифференциальные формы, скобки Ли, производные Ли... Смотрю и думаю: красиво же! Надо как-то использовать.
Но когда я открываю Ландафшица, я вижу безхитростное варьирование тензоров в индексной нотации. Физика стоит на принципе наименьшего действия, и варьировать приходится часто. Я не знаю, как выразить увиденное в стиле дифференциальной геометрии, и даже гугл не помогает.
Собственно, мне нужна помощь.
>> No.157326 Reply
>>157325
Физики всё равно будут писать всё в координатах. Ты их не переучишь. Хочешь заниматься физикой - делай как физики.
>> No.157328 Reply
>>157326
Ок, но я хочу уметь видеть не только как физик.
>> No.157339 Reply
File: Kvantui_Ya_znau.jpg
Jpg, 52.80 KB, 682×350 - Click the image to expand
edit Find source with google Find source with iqdb
Kvantui_Ya_znau.jpg
>>157325
Почитай Дубровина-Новикова-Фоменко. В первых двух томах есть разделы, посвящённые физике. И там вся машинерия вполне используется, хотя книга и устарела. Возьми какую-нибудь книжку по струнам, там используются гораздо более подвинутые вещи. Хотя иногда объясняются на простом языке.

А одно из самых простых и красивых применений в физике - уравнения Максвелла. В более простой формулировке A - 4-потенциал или 1-форма в пространстве Минковского. F = dA, напряжённость ЭМ поля. Первая пара уравнений Максвелла пишется как dF = 0, следует из того, что для дифференциальных форм d^2 = 0. Вторая пара следует из действия. Оно записывается так: S = \int (F, F) = \int F^F, где - звезда Ходжа. Варьируя в фиксированной калибровке легко увидеть, что вторая пара уравнений: dF = 0, если есть взаимодействие с материей, то получается dF = j, где j - ток. Электромагнитная дуальность - F -> F.

Чуть более продвинуто: A - форма связности в главном U(1)-расслоении, а F - кривизна. Неабелевы теории обобщаются именно так.
>>157326
физик физику рознь :3
>> No.157340 Reply
>>157339
>>157339
Кодировка поехала, формулы должны быть \int F^★F, где "★" - звезда Ходжа. Первая пара dF = 0, вторая пара d★F = 0 или d★F = ★j, если есть материя. Дуальность F -> ★F
>> No.157341 Reply
File: 65864.jpg
Jpg, 175.96 KB, 1280×828 - Click the image to expand
edit Find source with google Find source with iqdb
65864.jpg
Матанопроблемы.
1. Сначала не совсем из матана. Вот, например, мне нужно сформулировать отрицание сходящейся последовательности. Я не очень понимаю, отрицание меняет все кванторы или нужно думать о смысле в каждом случае?
Вот оригинальное определение: lim(n->inf)xn = a <=> A e>0, E ne, A n>ne: |xn-a|<e;
а отрицание должно выглядеть так, как я понимаю: последовательность хn не сходится <=> E e>0, A ne, E n>ne: |xn-a|>= e; То есть второй и третий кванторы не меняются. Так?
2. Следует ли сходимость последовательности xn из A p∈N E lim(n->inf)(x(n+p) - xn) = 0? (n+p) — это индекс. Почему? У меня есть вариант доказательства, но он не верен. Подозреваю, потому что не следует. Хотя, я исходил из критерия Коши, вот там |xm-xn|<e, если представить m как n+p, то |xm-xn| -> 0 => Ae |xm-xn| < e. Непонимат.
3. xn -> a+0 — это последовательность стремиться к а справа или слева? Есть f(x)->b, x->a+0, нужно определение сходимости для данной функции. Я так понимаю, оно будет таким: lim(x->a+0)f(x) = b <=> A e>0, E d>0, 0 < x-a < d: |f(x) - b| < e; так? Если бы было x->a-0, то было бы 0 < a-x < d; а если x->a? Или x->+inf, x->-inf, x->inf?
Для f(x) то же самое?
4. Пусть функция f неприрывна в точке a и в любой окрестности точки a существуют x1, x2 такие, что f(x1)*f(x2)<0. Докажите, что f(a)=0. У меня есть доказательство через теоремы Больцано-Коши, видимо оно не точное, что ли. За него стоит -+ (что в два раза хуже, чем +-). Хочу ваше доказательство.
5. Сформулировать определение функции, дифференцируемой в точке. Формулирую: f(x) — дифференцируема <=> можно записать виде f(x0+▲x)-f(x)=f'(x0)▲x +o(▲x). Или f(x) — дифференцируема <=> Elim(▲x->0)(f(x0+▲x)-f(x0))/▲x. Ошибки только в том, что слева от эквивалентности должно быть "f — дифференцируема в х0", а в первом случае справа "приращение ▲f можно записать в виде"? Больше нет?
6. Известно, что f(x) джважды дифференцируема на (1,7), f(2)=4? f'(2)=6, 3<=f''(x)<=4, x∈(1,7). a) Может ли f'(6)>15; б) Обязательно ли f(6)>55. Опять же у меня есть неверное доказательство того, что f'(6) не больше 15, где я опирался на то, что производная — это скорость роста функции. И есть идея проверять второе утверждение аналогично. Хотя бы скажите ответы типа "а — верно, б — неверно", хотя и доказательства хотелось бы посмотреть.
7. Чем плоха формулировка леммы о вложенных отрезках с педевикии, кроме того, что она не такая, какую давала препод? Или лемма о вложенных отрезках и теорема о вложенных отрезках и не одно и то же?
>> No.157342 Reply
>>157340
Красивый значок, кстати. Утащу и буду использовать :3
>> No.157343 Reply
>>157339
> Первая пара уравнений Максвелла пишется как dF = 0, следует из того, что для дифференциальных форм d^2 = 0
...потому, что мы как-то получили F = dA из A при варьировании действия. А как именно варьировать формы, и почему результатом получается именно то, что получается — неясно.
> Варьируя в фиксированной калибровке легко увидеть
И опять то же самое.
>> No.157344 Reply
File: Neighborhood_illu...
Png, 64.13 KB, 1057×1024
edit Find source with google Find source with iqdb
Neighborhood_illust1.svg.png
File: linear.png
Png, 14.62 KB, 387×312
edit Find source with google Find source with iqdb
linear.png

>>157341
> Я не очень понимаю, отрицание меняет все кванторы или нужно думать о смысле в каждом случае?
Думай геометрически. Вот у тебя есть точка a. Сходимость последовательнсти к a означает, что любая открытая (без границы) окрестность точки a содержит почти все точки последовательности. То есть, вне окрестности точки a лишь конечное число точек последовательности.
Соответственно, отрицание: существует такая окрестность, что бесконечно много точек осталось за бортом. Или, проще: сколько не стреляй, всё равно периодически попадаешь мимо.
> `∀p∈N ∃lim(n->inf)(x_(n+p) - xn) = 0`
Так как предел суммы это сумма пределов, это выражение можно немного упростить: ∃lim(n->inf) x_n.
А вообще, есть разница где ты задаёшь m и n: внутри ∀e ∃N или снаружи.
> xn -> a+0 — это последовательность стремиться к а справа или слева?
Справа. +0 же.
> Есть f(x)->b, x->a+0, нужно определение сходимости для данной функции.
`f(x)->b, x->a+0` если в пространстве `{x > a} f(x)->b, x->a`. По сути, всё то же самое, просто сначала мы отсекаем половину числовой прямой.
> Сформулировать определение функции, дифференцируемой в точке.
Функция дифференциируема в точке, если её можно линейно приблизить в этой точке. Т.е `f(x0+▲x) = f(x0) + f'(x0)▲x + o(▲x)`, да.
> Известно, что f(x) джважды дифференцируема на (1,7), `f(2)=4? f'(2)=6, 3<=f''(x)<=4, x∈(1,7)`.
> Может ли `f'(6) > 15`
Линейное приближение: `f'(6) = f'(2) + f''_max * (6 - 2) = 6 + 16 = 22`.
Собственно, да, может.
> Обязательно ли `f(6)>55`
Границей снизу будет `f(6) = f(2) + f'(2) (6 - 2) + f''_min (6 - 2)^2`. Считай сам, сколько это будет.
> Чем плоха формулировка леммы о вложенных отрезках с педевикии, кроме того, что она не такая, какую давала препод?
Понятие не имею, какую тебе давали. Попробуй доказать эквивалентность.
>> No.157345 Reply
>>157344
> Границей снизу будет
Возможно, я написал ерунду. Если что, приближай кусочно.
>> No.157350 Reply
>>157290
А новый форум хорена тебе как? Зарегистрировался там?
>> No.157353 Reply
>>157341
1. Допустим, у тебя есть строчка U, имеющая вид X1 X2 X3 ... Xn P, где каждый Xi - это квантор. Отрицанием U, то есть не-U, будет строчка Y1 Y2 Y3 ... Yn не-P, где каждый Yi - это квантор, двойственный к Xi.
Например, отрицанием строчки "∀x ∃y x+y>0" будет строчка "∃x ∀y x+y≤0".

Отрицание утверждения ты построил правильно. Но что значит "второй и третий кванторы не меняются"? Ты же их изменил.

2. Квантор можно навешивать только на переменную.
> A p∈N E lim(n->inf)(x(n+p) - xn) = 0
Это некорректная запись. Она вообще непонятно что означает. Соответственно, ответить на этот вопрос не могу.

3. Справа. Так, да - но только ты почему-то импликацию обозначаешь двоеточием.
Если бы a-0, то было бы ∀ε>0 ∃δ>0 : (0 < a-x < d) → |f(x) - b| < e.

4. Предположим, что речь идёт о функции f:ℝ→ℝ. Пусть f непрерывна в a.
Рассмотрим какую-нибудь систему стягивающихся окрестностей точки a. В каждой окрестности есть хотя бы одна точка, в которой значение функции положительно. Из каждой окрестности выберем одну такую точку и сформируем из них последовательность x1, x2, x3, ... , сходящуюся к a. Поскольку функция f непрерывна в a, соответствующая последовательность образов f(x1), f(x2), f(x3), ... сходится к f(a). Поскольку каждое из f(xi) положительно, f(a) ≥ 0. Далее, вернемся к нашей системе стягивающихся окрестностей. В каждой окрестности есть хотя бы одна точка, в которой значение f отрицательно. Из каждой окрестности выберем такую точку и сформируем из них последовательность y1, y2, y3, ... , сходящуюся к a. В силу непрерывности f, последовательность f(y1), f(y2), f(y3), ... сходится к f(a). Поскольку каждое f(yi) отрицательно, f(a) ≤ 0. Итак, f(a) ≥ 0 и f(a) ≤ 0 - что означает, что f(a) = 0.

5. Я не вижу ошибок.

6. Пример такой функции: f(x) = 4 + 6(x-2) + 2(x-2)^2. Он получается из ряда Тейлора, в котором остаток предположен нулевым.
f '(6) = 6 + 4(x-2) = 6 + 4(4) = 22.
f(6) = 4 + 6(2) + 2(4^2) = 4 + 12 + 32 = 48.
Ответ на а) да, ответ на б) нет.

7. Абсолютно ничем не плоха, всем хороша.
>> No.157355 Reply
>>157350
Нет. А разве этот форум реально сделали?
>> No.157360 Reply
>>157353
Спасибо.
> Отрицание утверждения ты построил правильно. Но что значит "второй и третий кванторы не меняются"? Ты же их изменил.
1. А, ну да, хех. Просто по ходу переделывал, что есть на листке, а когда то писал, вспомнил, что где-то видел утверждение типа "E ne, A n>ne — статичная конструкция, она не меняется при отрицании". Короче, кванторы всегда меняются?
2. Видимо, при записывании задания, я заменял слова "все" и "существует" на соответствующие кванторы.
3. > ты почему-то импликацию обозначаешь двоеточием.
Не знаю, принято так у нас, препод так пишет, хотя мне тоже сначала показалось странным.
Вот так верно будет: lim(x->-inf)=+inf <=> ∀E>0, ∃D>0, ∀x∈X, x<D: f(x)>E?
Еще, а если просто х->a? Тогда там 0<|x-a|<d будет?
4,5,6. Ясно.
7. Тогда так: можно ли записать "существует хотя бы одна точка c, принадлежащая всем отрезкам данной системы" как ∃c∈[a1,b1]ↄ[a2,b2]ↄ...ↄ[ak,bk]?
>> No.157362 Reply
>>157353
> Это некорректная запись. Она вообще непонятно что означает.
x(n+p) - xn можно интерпретировать буквально: yp,n = x(n + p) - x(n), и тогда при любом p последовательность yp,n → 0.
>> No.157368 Reply
>>157355
Да, вот буквально вчера.
>> No.157371 Reply
File: 366aud.jpg
Jpg, 124.71 KB, 600×631 - Click the image to expand
edit Find source with google Find source with iqdb
366aud.jpg
>>157343
Тут всё же не так. F=dA - это определение F, переменная интегрирования - A, поэтому первая пара уравнений в некотором смысле тавтологическая, что замечательно. Уравнения Эйлера-Лагранжа - это вторая пара. Теперь по поводу варьирования - это производная Лагранжиана по A. Лагранжиан L = 1/2 F^★F = 1/2(F, F) = 1/2(dA, dA) явно по определению. Теперь посчитаем вариацию (производную по A). δ(dA, dA)/2 = (δdA, dA) = (dδA, dA) = (δA, d^dA), где d^ = +-★d★ - сопряжённый d оператор. Отсюда получаем при невырожденности формы ★d★dA = 0 <=> d★F = 0. Вот вторая пара уравнений Максвелла без всяких индексных значков. При этом, на мой взгляд, использование координат/индексов в промежуточных вычислениях зачастую оправдано. Это бывает иногда очень простой и понятный способ получать инвариантные объекты.
>>157342
Мне тоже он нравится! Как и сама дульность Ходжа.
>> No.157372 Reply
>>157344
Странно, у меня из-за каких-то доброскриптов некоторые посты не отображаются, где они должны быть, а только внизу страницы ссылки на них и то не всегда. Вот и этот только сейчас появился.
>> No.157373 Reply
>>157360
1. Это даже нетрудно доказать индукцией по количеству кванторов.
Пусть U имеет вид "∀xP", тогда отрицанием U будет "∃x не-P".
Пусть U имеет вид "∃xP", тогда отрицанием U будет "∀x не-P".
Где в строчке P кванторов на одну штучку меньше.

3. lim(x->-inf)=+inf <=> ∀ε>0 ∃δ>0 ∀x∈X: x<-δ → f(x)>ε

7. Нет, так не пишут. Напиши ∃c: c∈⋂[ai;bi], i∈[1..k].

https://ru.wikipedia.org/wiki/Таблица_математических_символов
https://ru.wikipedia.org/wiki/Греческий_алфавит
>> No.157374 Reply
>>157373
Ясно.
>> No.157377 Reply
Я пометался по интернету и не смог в первой выдачи найти доказательство того, что порядок элемента делит порядок группы, кроме как через теорему Лагранжа. А можно ли еще проще?
>> No.157381 Reply
File: F-=-dA.jpg
Jpg, 558.92 KB, 1429×2204 - Click the image to expand
edit Find source with google Find source with iqdb
F-=-dA.jpg
>>157371
> F=dA - это определение F
А F мы откуда взяли? А F мы взяли из пикрелейтеда.
Изначально у нас есть только 4-потенциал.
> Отсюда получаем при невырожденности формы ★d★dA = 0 <=> d★F = 0
Неплохо. Надо попробовать нечто подобное с дополнительным членом вида A^★j.
>> No.157385 Reply
>>157377
Порядок элемента - по определению порядок порождённой этим элементом подгруппы. Утверждение, что порядок некоторых подгрупп делит порядок группы, проще всего доказать, доказав, что порядок любой подгруппы делит порядок группы.
>> No.157411 Reply
File: snapshot3.png
Png, 143.91 KB, 1260×823 - Click the image to expand
edit Find source with google Find source with iqdb
snapshot3.png
Что обозначено подчеркнутым и надчеркнутым х? Не очень понимаю, что написано после "Вследствии теоремы Больцано-Вейерштрасса".
>> No.157415 Reply
File: 14840094767810.jpg
Jpg, 23.67 KB, 480×360 - Click the image to expand
edit Find source with google Find source with iqdb
14840094767810.jpg
>>157381
F - это удобное обозначение. Мы стартовали с 1-формы A и хотим написать кинетический член. Кнетический член бозонного поля должен быть "квадратом производной поля". Для 1-формы естественная производная- внешний дифференциал F=dA, где F - это просто удобное обозначение, тем более, что в координатаъ dA громоздко записывается как (dm An - dn Am). Квадрат производной (dA, dA) = F^★F, так и строится теория.

С дополнительным членом, а именно с источником, ещё проще - (A, j) = A^★j, вариация - дифференциал δ. δ[ (dA, dA)/2 - A^★j] = d^✴ F - ★j = 0

Капча: успехов грустным фапающий прежней
>> No.157416 Reply
>>157415
> Кнетический член бозонного поля должен быть "квадратом производной поля". Для 1-формы естественная производная- внешний дифференциал F=dA
Лётчик.жпг
> С дополнительным членом, а именно с источником, ещё проще
Это я и так уже понял.
>> No.157429 Reply
File: sad_reinhardt_by_zinph1212-da89rcm.jpg
Jpg, 120.05 KB, 1024×685 - Click the image to expand
edit Find source with google Find source with iqdb
sad_reinhardt_by_zinph1212-da89rcm.jpg
Завтра писать экзамен, не поможете подготовиться? Я пытаюсь порешать задачек для подготовки, но многие не даются.
> Группа ортогональных 3х3-матриц с определителем 1 гомеоморфна RP^3.
Нагуглил, что для этого, видимо, полезно построить двулистное накрытие S^3 -> SO(3), но и его придумать не смог...
> Построить непрерывное отображение T^2 = S^1xS^1 -> S^2 ненулевой степени, вычислить его степень.
Степень тут, как я понял, это коэффициент, на который умножается образующая Z при отображении Z=H2(T^2) -> H2(S^2)=Z, индуцированном на гомологии от данного. Как построить само отображение?
> Г - граф из двух вершин соединенных тремя ребрами. Найти всего его двулистные накрытия с точностью до эквивалентности накрытий.
От меня хотят накрытие букета двух окружностей? У него же оооочень много всевозможных накрытий...
> Всякое непрерывное отображение линейно связного топологического пространства, имеющего конечную фундаментальную группу, в S^1 гомотопно постоянному.
Вообще непонятно как это всё связано...
> Пример конечнократного накрытия связного симплициального комплекса X связным комплексом Y, такого, что гомологии пространств X,Y с коэффициентами в Z_2 не изоморфны.

Буду очень благодарен за помощь с любой из задачек этих.
>> No.157431 Reply
>>157429
Времени мало, отвечу кратко.
Двулистное накрытие SU(2) -> SO(3). У них одинаковая алгебра Ли, (в su(2) базис матриц i \sigma/2, \sigma - матрицы Паули). При этом экспонета матрицы паули на угол 2 pi = минус единичной матрице, то есть накрытие двулистное.

Это также число прообразов общей точки со знаком (ориентацией). Пример отображения - фактор тора по Z/2. Тор - фактор плоскости со склеенными сторонами. Z_2 - умножение на -1. У него 4 неподвижных точки, накрытие ветвится в них, получается сфера с четырьмя защипами - "подушка". Вроде степень не 0.

Ты сам написал,что накрытия двулистные

Если фундаментальная группа конечна, то любая петля в какой-то степени тривиальная, то есть в пространстве нет петель, чтобы зацепиться за окружность.

S^2 накрывает RP^2, у последнео первые гомологии равны Z^2, ну или SU(2)=S^3 накрывает RP^3
>> No.157434 Reply
>>157431
C первой и последней разобрался
> число прообразов общей точки со знаком (ориентацией)
Общей это какой? Я так понял, нужно нарисовать на плоскости квадрат, который мы позже склеим в тор, подействовать на него группой Z_2 умножая координаты на -1, тогда 4 угла квадрата будут переходить друг в друга, что в торе одна точка. Дальше совсем непонятно.

Всё остальное тоже ещё не понял, но в любом случае спасибо тебе за ответ! Эх, иметь бы умение задачки хорошо решать...
>> No.157435 Reply
>>157429
О, придумал третье, как граф накрывать. Смотрим на окрусность вокруг общей точки: крестик. Делаем две его копии, которые перейдут в эту окресность, а дальше соединяем его концы в разных комбинациях с расчетом на то, чтоб сохранить направление каждого пути. Всего получается 3 таких накрытия.
>> No.157437 Reply
Продолжая тему задачек по топологии, как вычислить гомологии произведения CP^2xS^1? По отдельности я и то, и другое считать умею, но что с произведением делать... А ещё, если я хочу доказать, что sin(4pix)cos(6piy) морсовская на торе, что я делаю: дифференциал нахожу, потом x,y на квадрате [0,1]*[0,1], которые его зануляют, а дальше? Это я нашел особые точки, а чтоб они были морсовскими ведь ещё нужно чтоб в окресности каждой были локальные координаты такие чтоб функция в ней была суммой значения в особой и квадратов координат. Я думал в тейлора разложить, но мне кажется это уж совсем странно делать было бы...
>> No.157440 Reply
>>157437
когомологии произведения - произведение когомологий (формула Кюннета). Морсовская - достаточно проверить, что особенность невырожденная. Локальные координаты тогда гарантируется леммой Морса. Невырожденная - матрица вторых производных невырожденная. >>157434
>>157434
Общая точка - это не специальная :) (не в смысле теории схем!) А именно есть конечное число точек, в которых что-то плохое (ветвление, меньше прообразов), а все остальные - общие. Примерно так. Только нарисуй квадрат с вершинами (0, 0) (0, 1) (1, 0) (1, 1) и отождествляй с периодически повторяющимися. Умножаешь всё на -1. Тогда у тебя получается сфера! Неподвижные точки - 4 угла, то есть одна точка, ещё середины рёбер ( две точки после склейки) и центр квадрата (ещё одна точка) - всего 4 после склейки. А у остальных точек прообраза будет два. Представь, во что это всё склеится

Желаю удачи на экзамене!
>> No.157442 Reply
File: overwatch__reinhardt___mercy_by_haje714-da4bwn6.jpg
Jpg, 129.11 KB, 1024×754 - Click the image to expand
edit Find source with google Find source with iqdb
overwatch__reinhardt___mercy_by_haje714-da4bwn6.jpg
>>157440
Получается, идея такая: я вычисляю гомологии каждого пространства, через них нахожу когомологии по двойственности Пуанкарэ, беру их произведение и нахожу из него когомологии произведения, а из них опять по Пуанкарэ (не уверен, что он будет для произведения работать) нахожу гомологии произведения?

Про морсовскую - вот помнил ведь смутно, что говорили проверять невырожденность гессиана!

Спасибо тебе за помощь, анончик! До экзамена ещё часов 6, постараюсь за это время остальное понять.
>> No.157456 Reply
Я в качестве эксперимента попробовал создать сами-знаете-где филиал этого треда. Посмотрим, что из этого выйдет. Ссылку вкидывать не буду, дабы не провоцировать, но кто захочет — тот найдет, ведь так?

к: поддержать
>> No.157458 Reply
File: reinhardt_by_iamsum-da3afb8.png
Png, 479.90 KB, 1024×1027 - Click the image to expand
edit Find source with google Find source with iqdb
reinhardt_by_iamsum-da3afb8.png
Экзамен написал, решил половину задачек. Они оказались проще, чем то, на чем я готовился. Спасибо за помощь вчерашнюю, доброанон :3 Ты меня как минимум морально очень поддержал, уже не так страшно было идти сдавать хоть что-то поняв из курса. Пора начать серьезнее относиться к математике, а то так и буду как тварь дрожащая 4 сессии в год ещё несколько лет.
>> No.157459 Reply
>>157371 >>157415
Не сходится. Там множитель 1/16PI должен был превратиться в 1/4PI, т.е не хватает ешё одной двойки.
>> No.157475 Reply
File: ZrHgbsEKTHg.jpg
Jpg, 100.89 KB, 800×600 - Click the image to expand
edit Find source with google Find source with iqdb
ZrHgbsEKTHg.jpg
>>157458
Рад помочь! Математика - это дело хорошее.
>>157459
Это нормировка, она ни на что не влияет, подели действие на два, или я не понял, что не сходится. Когда пересчитываешь инвариантное выражение в координатах надо помнить, что если ты суммируешь F{\mn} = dm An - dn A_m по всем (m, n), то тебе надо умножить на 1/2, чтобы получить правильное dF, ну и у звезды Ходжа ★ есть своя нормировка.
>> No.157476 Reply
>>157475
Ух ты, моя картинка. Приятно.
>> No.157477 Reply
File: вторая-пара.png
Png, 91.14 KB, 1404×2196 - Click the image to expand
edit Find source with google Find source with iqdb
вторая-пара.png
>>157475
Пикрелейтед. Мы начали с 1/16π F индексы F индексы, а пришли к 1/4π дэ F индексы по дэ x индексы.
>> No.157542 Reply
1=sqrt((-1)(-1))=sqrt(-1)sqrt(-1)=i^2=-1
че не так?
>> No.157543 Reply
>>157542
Корень из отрицательного числа - многозначная функция. sqrt(-1) = ±i.
Поэтому √(ab) ≠ √a × √b.
>> No.157544 Reply
File: not-true.png
Png, 34.15 KB, 1342×269 - Click the image to expand
edit Find source with google Find source with iqdb
not-true.png
>> No.157545 Reply
>>157543
>>157544
Но ведь в той же школе кучу раз делал именно так, как вы говорите, только наоборот. Когда и где эти правила были оговорены? Я имею в виду картинку.
>> No.157546 Reply
>>157545
В школе ты работал только с положительными вещественными числам. Для положительных вещественных чисел √(ab) = √a × √b. Для всех остальных чисел это равенство не выполняется. Ты толстячок, да?
>> No.157547 Reply
File: E_dbmWXdwTA.jpg
Jpg, 10.64 KB, 274×292 - Click the image to expand
edit Find source with google Find source with iqdb
E_dbmWXdwTA.jpg
>>157546
Нет, я реально не помню никаких таких ограничений. Вообще мне знакомый пикрил скинул. И я не понимаю, где ошибка. А думая об этом, пришел к той фигне, что скинул вам.
>> No.157549 Reply
>>157547
Тебе объяснили, где ошибка.
√(ab) ≠ √a × √b.
1/√a ≠ √(1/a).
>> No.157554 Reply
Допустим, есть видео 1280x960 8bit 12 fps - это площадь изображения в 1228800 пикселей, помноженная на глубину цветности в 8 бит, что равно 9830400 бит, то-есть около 8 мегабит или 1 мегабайта по 12 раз в секунду, получается 96 мегабит или 8 мегабайт в секнуду. Такой поток получается без сжатия, а вот как какой кодек ужимает видео, насколько он делает его меньше в результате, я понять не могу, потому что в статьях или рассказывают, как применять кодеки, либо разжёвывают, что кодеки делают/могут делать, но я не могу понять, как те манипуляции влияют на битрейт выходного видео. Может, что-то тут знает, что значат все эти:
https://ru.wikipedia.org/wiki/H.264
логарифмические квантования с предсказаниями?
>> No.157555 Reply
>>157549
Но в какой строке конкретно это нарушают? Я этого нигде не вижу. Единственную ошибку, которую я вижу - это то, что во второй строке у нас получается i=-i, но это тоже как-то не подходит под те правила, что ты скинул.
>> No.157556 Reply
>>157555
Между второй и третьей, например.
>> No.157559 Reply
>>157547
из √{-1/1} = √{1/-1} не следует √{-1}/√{1} = √{1}/√{-1}. Более того, второе неверно; i/1=i ≠ 1/i=-i. А далее из ложного утверждения, как известно, выводится всё что угодно.
>> No.157566 Reply
Не хватает мощности мозга, чтобы осилить понятия решения интегральных уравнений.
Например, не понимаю что такое резольвента.
И не могу увидеть как решить второй час уже, например:
Ф(х)=х+Se^(x-t)ф(t)dt
Через сведение к диффуре.
Как прокачаться? Посоветуйте.
>> No.157567 Reply
>>151715
Доброй ночи доброанон. Решил заняться математикой с самого детства она мне нравилась С чего начать? Есть что-нибудь довольно всеобъемлющее, так сказать с чего можно начать ознакомление и составить более целостное представление?
>> No.157568 Reply
>>157567
А ты уже прочитал >>124265?
>> No.157572 Reply
>>157547
Даже так можно сказать, смотря как понимать корень. Квадратных корня у числа всегда два. И, например, при переходе к последней от предпоследней нужно выбрать -1=sqrt(1)
>> No.157577 Reply
>>157572
Радикалом все же обозначается только наименьший по фазе корень.
>> No.157579 Reply
Ребят, был экзамен по матану, в билете у меня была такая штука, что функция, имеющая производную в какой-то точке непрерывная в ней, и был вопрос, а что если производная в точке равная бесконечности. Следует ли из этого, что функция непрерывна в точки? Если да, то доказать, что это верно всегда, если нет, то привести пример
И функция должна быть определена в точке, которую рассматриваем
Потому что я сначала сразу же написал 1/abs(x)
И типа производная в 0 - бесконечность, функция разрывна. А он сказал, что это не то потому, что функция там не определена.
Вопрос, как же тогда правильно на это ответить? И что за бред с его замечанием? Разве то, что функция неопределена в точке не значит, что она там разрывна? Это же одно и то же. Определена - неразрывна, неопределена - разрывна. Нет?
>> No.157580 Reply
>>157579
Это совершенно разные показатели этого параметра. Неопределённость функции в какой-либо местности лишь открывает возможности дальнейшего анализа неопределённого участка на предмет отсутствия или же присутствия и дальнейшего исследования полученных данных.
Молодой человек, к математике, действительно, стоит подходить с порядком в голове; чем точнее разбираетесь - тем лучше.
>> No.157581 Reply
Прошу прощения.
>>157580
Это совершенно разные показатели параметра (нахождения, существования). Неопределённость функции в какой-либо местности лишь открывает возможности дальнейшего анализа неопределённого участка данной функции на предмет её присутствия в данной местности и дальнейшей обработки полученных данных.
>> No.157584 Reply
>>157579
> Определена - неразрывна, неопределена - разрывна. Нет?
Нет. Функция Хевисайда определена всюду, но не непрерывна. 1/abs(x) можно доопределить нулём в нуле, со всеми профитами вроде тру непрерывности.
>> No.157585 Reply
>>157579
Я от матана далек, но что если доопределить твою функцию в этой точке нулем?
>> No.157586 Reply
>>157585
Не увидел поста выше, извиняюсь.
>> No.157615 Reply
>>157580
>>157581
Как так. Если функция определена в точке x0, значит там есть её значение f(x0) а значит lim(x->x0)[f(x)]=f(x0) и функция непрерывна в этой точке. Можете привести пример определенной везде, но разрывной функции? И без доопредлений.
>>157584
Википедия:
> > Функция Хевисайда — кусочно-постоянная функция, равная нулю для отрицательных значений аргумента и единице — для положительных. В нуле эта функция, вообще говоря, не определена...
Опять доопределения,не годится.

>>157577
Так что с моим вопросом и как на него правильно ответить?
>> No.157616 Reply
>>157615
> И без доопредлений.
По религиозным причинам?
>> No.157617 Reply
>>157616
Нет, просто думаю, что доопределения моего препода не удовлетворили бы.
>> No.157618 Reply
>>157617
Любая функция лишь отображает число (или набор чисел) в число, то, как именно она задана — неважно. Доопределение ничем не хуже других способов задать функцию.
>> No.157636 Reply
File: 1.jpg
Jpg, 31.00 KB, 500×375 - Click the image to expand
edit Find source with google Find source with iqdb
1.jpg
Анон, собираюсь вспомнить и выучить заново школьную программу по математике.
Посоветуй, пожалуйста, с чего вообще начать, какие материалы использовать? Всё уже забыл 21 год, но нужно поступать в вуз.
Это пока не подготовка к ЕГЭ, просто хочу подтянуть знания.

Реально ли вообще темы от 4 до 11 класса пройти за год?
>> No.157637 Reply
>>157636
> Реально ли вообще темы от 4 до 11 класса пройти за год?
Месяца три, не больше. И это еще с большим запасом.
>> No.157638 Reply
File: Untitled.png
Png, 4.96 KB, 462×179 - Click the image to expand
edit Find source with google Find source with iqdb
Untitled.png
>>157637
Это учитывая затворничество?
Скоро из свободного времени будет 4 часа в лучшем случае.
>> No.157639 Reply
>>157638
Нет, это на изи-моде, 4 часа более чем достаточно. Вся школьная математика охватывает совсем мало тем (дроби, многочлены, рациональные функции, степенные функции, пределы, производная, тригонометрия, что я еще забыл?), и каждая из них может быть на школьном уровне полностью освоена за неделю. Надрачивание на типовые задания, чтобы их решать быстро требует побольше времени, но это тебе вроде пока не нужно. Затык может случиться с геометрией, и особенно стереометрией, но в остальном все очень просто. Никогда не понимал, как они это умудряются растягивать на 11 лет ах да, надрачивание типовых задач на скорость.
Хинт: сразу шли школьную тригонометрию со всеми формулами синусов тройной суммы нахуй, лучше почитай ее связь с комплексными числами и выводи все оттуда.
>> No.157640 Reply
>>157639
Спасибо.
> Надрачивание на типовые задания, чтобы их решать быстро
А это ещё зачем?
Энивей, сейчас это не нужно, да.
>> No.157641 Reply
>>157640
> А это ещё зачем?
Для ЕГЭ. До этого - для экзаменов, олимпиад, контрольных. В этом на самом деле есть определенный смысл, ибо для того чтобы знания закреплялись, их нужно постоянно использовать. Так что не забивай на решение задачек после того как пройдешь весь курс, иначе опять всё забудешь.
>> No.157650 Reply
>>157636
Няша. Ты это я. Почему не подготовка? Когда сдавать хочешь, куда поступать?
>>157639
Ненавижу голое «надрачивание на типовые задания», в итоге никаких знаний, только легко забывающиеся механические навыки. Сдал и забыл. И скука смертная.
>> No.157653 Reply
>>157650
Поступать хочу через год-два, а сейчас нужно уже начинать работать, итак пару лет на шее родителей проебал.
Для интереса можно и сейчас попробовать задачки с ЕГЭ порешать, когда задрочу школьную программу.
К слову, в школе я учился не оче хорошо, так что вспоминать буду долго.
> куда поступать?
На технаря :3 в какой-нибудь мухосранский вуз, возможно и в ДС попробую перебраться.
c:успехом научишься
>> No.157659 Reply
>>157653
В ДС однокомнатная квартира стоит 7,5 миллиона ₽. Зарплата 25000₽. 300 месяцев или 25 лет работы не кушая, не одеваясь, не платя за жкх, не кормя и одевая детей. Прикить ещё кредитные проценты. Прикинь ещё форсмажор как то укрепление рубля и снижение зарплат, при стабильной сумме кредита в рублях. Причём после 25 лет выплаты за квартиру она уже не нужна ибо детородный возраст вышел. Вот потому моя Москва и есть город переселенцев, потому что местные вымирают довольно быстро.
>> No.157660 Reply
>>157659
> Прикить
Прикиньте
>> No.157663 Reply
>>157659
Берёшь ипотеку на 500 лет и живёшь.
>> No.157664 Reply
>>157641
Глупый вопрос.
Где найти точный список разделов математики, представленных на ЕГЭ
>> No.157679 Reply
>>157664
Вместе с демовариантами публикуют спецификации. Это то, что тебе нужно.
>> No.157681 Reply
File: 6zqay1.jpg
Jpg, 84.96 KB, 600×800 - Click the image to expand
edit Find source with google Find source with iqdb
6zqay1.jpg
>>157663
> Берёшь ипотеку на 500 лет и живёшь.
Даже Дункан Маклауд, с Кощеем Безсмертным столько не прожили!
>> No.157682 Reply
>>157681
не оффтопь, парниша
>> No.157683 Reply
>>157681
Кощей прожил.
>> No.157685 Reply
>> No.157695 Reply
>>157685
Тебе же дали контрпример.
>> No.157699 Reply
>>157685
Твой препод дал отворот-поворот твоей функции не из-за доопределения, а из-за того, что пример был неверный. Тебе выкатили очевиднейший ответ, немного поправив твой. В чем проблема?
>> No.157709 Reply
Как доказать что единственной нормальной подгруппой А4 является V4? Желательно без теорем Силова.
>> No.157711 Reply
File: GroupDiagramMiniA4.svg.png
Png, 43.01 KB, 1024×1024 - Click the image to expand
edit Find source with google Find source with iqdb
GroupDiagramMiniA4.svg.png
>>157709
В А4 не так много элементов, можно даже тупо перебором.
>> No.157712 Reply
>>157711
Найти нормальные группы перебором? Я не знаю как его оптимизировать.
>> No.157713 Reply
>>157711
Я так понимаю на картинке в центре тождественна перестановка, 3 ветви внизу – это обратные сами себе перестановки из V4, а 4 цикла – это перестановка, обратная ей и тождественная.
>> No.157714 Reply
>>157709
Руками перебрать.
>> No.157715 Reply
>>157714
Это как? Взять все пары элементов, а их 12*11. потом среди этого числа как-то найти группу, такую что она переходит только в себя?
>> No.157717 Reply
File: класс1.PNG
Png, 155.38 KB, 688×601
edit Find source with google Find source with iqdb
класс1.PNG
File: класс2.PNG
Png, 179.78 KB, 694×833
edit Find source with google Find source with iqdb
класс2.PNG

>>157715
Перебирать нужно классы сопряженности, конечно. Подгруппа нормальна тогда и только тогда, когда является объединением классов сопряженности.
>> No.157718 Reply
>> No.157719 Reply
>>157717
Спасибо.
>> No.157720 Reply
>>157713
Да, это граф циклов.
>> No.157724 Reply
>>157695
Не вижу никакой примера, кроме функции Хевисайда, но я на 1 курсе, мы такое не проходили, так что не подходит. Так что вопрос все ещё открыт.
>>157699
Не вижу никакого ответа.
Если бы я дал как пример f(x)={-1/x, x!=0 и 0, x=0} это бы было правильно? Можно ли дать ещё какой-нибудь ответ без доопределений? Вообще хотелось бы увидеть ещё примеры функций, которые определены, но прерывны в какой-то точке. Без доопределений.
>> No.157725 Reply
>>157724
f(x)={-1/x, x!=0 и 0, x=0}
Да, вот такой пример подходит и именно его тебе предъявили выше.
Определись, то ли ты на первом курсе и хочешь элементарный пример. Или ты хочешь функцию задаваемую одной формулой. Разумеется такая функция есть, но в она точно не выражается в определениях даваемых в школе и на первом курсе.
>> No.157739 Reply
Короче, вопрос уровня /u: есть парень, 21 год, ему надо подняться в математике на уровень школьной программы за полгодика, именно столько осталось до экзаменов. Начинать надо с азов, к примеру, всякие задачи на движение объектов, которые решаются с помощью линейных уравнений охватить нужно. Посоветуйте литературы, заточенной под такие-то цели. Я буду ему помогать, со школьной математикой у меня всё хорошо.
>> No.157748 Reply
>>157739
По-видимому, школьные учебники + Сканави.
>> No.157776 Reply
>>157748
А какие школьные учебники посоветуете? Я не знаю, какие хорошие.
>> No.157777 Reply
>>157776
Алгебра - Макарычев, потом Колмогоров. Геометрия - Погорелов с Атанасяном.
>> No.157796 Reply
>>157777
Геометрия: потом Колмогоров. По алгебре лучше сначала Шень.
>> No.157861 Reply
Нужен совет, вдруг кто-то тут проходил через такое же... В общем, у меня есть фобия математики. Очень странная фобия: мне нормально учить теорию, обычно я её легко понимаю на среднем уровне, а если приложить реальные усилия то и хорошо выучить могу что угодно. Тем не менее, я боюсь решать задачи. Даже начинать страшно: кажется, что если я начну, то потеряю 4-5 часов впустую, почувствую себя умственно отсталым, будет от этого больно и на этом все закончится. Причем нет осознанного страха, как перед гопником с ножом. Просто я заметил, что всеми правдами и неправдами откладываю решение ("ну поботаю ещё теорию, я ещё не готов задачи решать"; "ну я сегодня уставший, лучше завтра с утра начну"), легко бросаю, если что-то не идет. При этом я отлично понимаю, что нельзя научиться решать задачи не решая задачи, что нет никакой царской дороги вокруг и нужно просто брать и делать. Причем я беру и нормально делаю, и не страдаю даже если не получается, когда под всякими веществами (для интереса пробовал под 4 разными, наверное). Просто страх уходит и я спокойно, не загоняясь пытаюсь решать. Если не получается , то спокойно думаю почему, что нужно найти, чтоб получилось. В общем, именно так я и понял, что главный мой враг это страх. И даже решив кучу задачек, я боюсь идти их сдавать. И я понимаю, что без научной статьи единственный способ поступить в хорошую магистратуру/на PhD, это иметь хорошие оценки и рекомендации, а чтоб были хорошие оценки НУЖНО ехать и сдавать то, что решил. Надеюсь, кто-то из вас знает, как с таким бороться...и
>> No.157871 Reply
>>157861
Анон, это психологические проблемы, не математические. Спрашивать о таком тебе нужно не у математиков.
>> No.157872 Reply
>>157861
Простые задачи решать не обязательно, достаточно просто понять, как их можно решить.
В любом нормальном учебнике задач, которые можно решить, просто посмотрев на них, полно. Просто начинай всегда с них.
>> No.157874 Reply
>>157872
Да, и не обязательно садиться решать задачу, можно просто переодически думать о том, как её решить.
>> No.157875 Reply
>>157874
Это не работает. После того как подумал, надо всё равно сесть и честно расписать.
>> No.157877 Reply
>>157875
Это работает, потому что после достаточного обдумывания у тебя возникнет острое желание
> сесть и честно расписать
безо всяких «надо».
>> No.157880 Reply
>>157872
Идея хорошая, но нам в вузе дают листки с задачами. Там часто бывает простых задач вообще ни одной нет. Ко многим даже не знаешь как подступиться, хотя и определения все знаешь, и теоремы из курса какие-то помнишь...
>>157877
Тоже очень разумно звучит. Поставлю целью в метро или ещё где-нибудь в пути всегда думать хоть о какой-то задаче. Всё равно там заняться нечем.

Если будут ещё какие-то идеи, то я буду очень за них благодарен. За последний год я очень продвинулся в борьбе с фобией этой, но она до сих пор не сдается и мешает хорошо учиться. Кстати, до поступления в вуз ничего такого не было: я в любую задачу вгрызался, даже когда мне прямо говорили, что она "слишком сложная для меня".
>> No.157883 Reply
Недавно у меня на Харкаче состоялся разговор, где некий аноним утверждал, что "математически умные люди не то же самое, что общепринято умные люди", в контексте толстого вброса про талант и врождённые способности. Как это понимать?
>> No.157886 Reply
>>157883
Насчет второй части фразы я тебя не понял, но первая половина верная. Знаю огромное количество математически одаренных, но не умеющих в эту жизнь, хотя просто умных людей среди математиков намного больше. В общем мат сообщество делится на две противоположности.
>> No.157890 Reply
File: Безымок.png
Png, 9.93 KB, 545×157 - Click the image to expand
edit Find source with google Find source with iqdb
Безымок.png
>>157861
> мне нормально учить теорию, обычно я её легко понимаю на среднем уровне, а если приложить реальные усилия то и хорошо выучить могу что угодно.
"Человек может все, пока ничего не делает".
Это обычная мантра для снижения тревожности в рамках механизма прокрастинации. Точно такая же, как и:
> поботаю ещё теорию, я ещё не готов задачи решать
> ну я сегодня уставший, лучше завтра с утра начну
На самом деле ты не можешь хорошо выучить что угодно - люди конечны, ресурсы ограничены, у каждого есть свой интеллектуальный потолок. Твоя самооценка завязана на успех, а понятие успеха завязано на академические достижения. Последние же, как тебе известно, во многом зависят от уровня интеллекта. Каждая нерешенная задача (или ошибочно решенная задача, или неточно решенная задача, или медленно решенная задача) ставит под сомнение твои умственные способности - и самооценка оказывается под угрозой. Неудивительно, что ты пытаешься избежать проб, тестов, ревизий и проверок на вшивость. Если твои реальные способности окажутся ниже предполагаемых (а это так, это всегда так, это справедливо для любого человека) - в твоем мире это будет катастрофой. Умник больше не умник, эрудита уличили в некомпетентности, вероятность достижения цели - ноль, самооценка - ноль, пыщь-пыщь отказ системы.
> Кстати, до поступления в вуз ничего такого не было: я в любую задачу вгрызался, даже когда мне прямо говорили, что она "слишком сложная для меня".
Ну да, ведь рядом не было конкурентов и казалось, что до успеха рукой подать. А теперь за успехи особо не хвалят, а каждое неправильное решение может привести к медному тазу над магистратурой - а без успешной академической карьеры ты нуль, грязь, вошь, пыль, тупица, васян и пустое место.

Ты тревожник. Тревожник живет не в мире вещей, а в мире будущих угроз. У тебя катастрофизация работает в обе стороны - ты переоцениваешь как негативные, так и позитивные последствия событий. Ты когда эти задачки сдавать несешь, ты ведь чуть ли не жизнь на карту ставишь - ты только подумай, насколько комично это выглядит со стороны. Если твои страхи определяют твое поведение и это продолжается больше полугода, тебе пора к психотерапевту - и это шутка и не фигура речи.
>> No.157891 Reply
>>157890
Извини, анон, но ты хуйню написал.
>> No.157892 Reply
Психологическая математика и математическая психология. Только сегодня, только в нашем треде.
>> No.157893 Reply
>>157890
> На самом деле ты не можешь хорошо выучить что угодно - люди конечны, ресурсы ограничены, у каждого есть свой интеллектуальный потолок
На самом деле, это вопрос времени. На некоторые теории и задачи просто нужно куда больше времени, чем хотелось бы.
>> No.157895 Reply
>>157891
Значит, я не прав, и на самом деле мы имеем дело с редчайшим случаем математической фобии - уникальным, неповторимым случаем, достойным вписания в анналы. Боязнь математики, ага. Можете бояться своих задачек дальше, мальчики, я не против.
>>157892
Не надо иронизировать. Когда человек пытается выдать проблемы с головой за проблемы с матаном - у него действительно... ммм... проблемы с головой.
>>157893
На самом деле:
1)Время и есть ресурс, причем один из основных, если не самый основной. Чтобы освоить некоторые теории и задачи, скоро будет мало целой жизни (да уже мало).
2)Никакая куча времени не компенсирует низкий интеллект и скорость обучения. Человек с IQ<90 может до старости сидеть над банальным матаном и все равно не поймет даже основ, а обезьяна не осилит этих основ даже за вечность. Если ты думаешь, что человек способен осилить все, что угодно, ты очень сильно переоцениваешь человеческие способности - и свои тоже.
>> No.157897 Reply
>> No.157902 Reply
>>157895
Явление достаточно распространненое: https://en.wikipedia.org/wiki/Mathematical_anxiety
А вот у тебя, судя по этому посту >>157890, какое-то шизоидальное расстройство. Желаю тебе прожить с ним всю жизнь :3


Password:

[ /tv/ /rf/ /vg/ /a/ /b/ /u/ /bo/ /fur/ /to/ /dt/ /cp/ /oe/ /bg/ /ve/ /r/ /mad/ /d/ /mu/ /cr/ /di/ /sw/ /hr/ /wh/ /lor/ /s/ /hau/ /slow/ /gf/ /vn/ /w/ /ma/ /azu/ /wn/ ] [ Main | Settings | Bookmarks | Music Player ]